Будь умным!


У вас вопросы?
У нас ответы:) SamZan.net

Министерство образования и науки Российской Федерации

Работа добавлена на сайт samzan.net:


Министерство образования и науки Российской Федерации

Министерство образования и науки Российской Федерации

Федеральное государственное автономное образовательное учреждение высшего профессионального образования

«Российский государственный профессионально-педагогический университет»

Машиностроительный институт

Кафедра общей химии

ЗАДАНИЯ И МЕТОДИЧЕСКИЕ УКАЗАНИЯ

К ВЫПОЛНЕНИЮ КОНТРОЛЬНЫХ РАБОТ

ПО ДИСЦИПЛИНЕ «ХИМИЯ»

для студентов всех форм обучения

направления подготовки  051000.62  
Профессиональное обучение (по отраслям)

профилей подготовки  «Информатика и вычислительная техника», «Энергетика», «Правоведение и правоохранительная деятельность», «Экономика и управление», «Транспорт»,
«Машиностроение и материалообработка», «Металлургия»;

направления подготовки  230400.62 Информационные системы и технологии

профиля подготовки «Информационные технологии в медиаиндустрии»;

направления подготовки 140400.62 Электроэнергетика и электротехника

профиля подготовки «Электрооборудование и электрохозяйство предприятий, организаций и учреждений»

Екатеринбург

2012


Задания и методические указания к выполнению контрольных работ по дисциплине «Химия». Екатеринбург: ФГАОУ ВПО «Рос.гос.проф.-пед.  университет», 2012. 72 с.

Задания и методические указания к выполнению контрольных работ по дисциплине «Химия» предназначены для студентов всех форм обучения направления подготовки  051000.62 Профессиональное обучение (по отраслям) профилей подготовки  «Информатика и вычислительная техника», «Энергетика», «Правоведение и правоохранительная деятельность», «Экономика и управление»,  «Транспорт», «Машиностроение и материалообработка», «Металлургия»,  направления подготовки  230400.62 Информационные системы и технологии профиля подготовки «Информационные технологии в медиаиндустрии» и  направления подготовки 140400.62 Электроэнергетика и электротехника профиля подготовки «Электрооборудование и электрохозяйство предприятий, организаций и учреждений».

Они включают основной теоретический материал по всем темам курса химии, примеры решения задач, контрольные задания и необходимые справочные данные.

Составители:  доцент, канд. хим. наук Слинкина М.В.

      доцент, канд. хим. наук Харина Г.В.

Одобрены на заседании кафедры общей химии. Протокол от
04.09. 2012. № 1.  

Заведующий кафедрой ОХ      Н.Т. Шардаков

Рекомендованы к печати методической комиссией Машиностроительного института РГППУ.   Протокол от 19.09.2012.  № 1.

Председатель методической

комиссии  МаИ РГППУ       А.В. Песков

©ФГАОУ ВПО «Российский

государственный

профессионально-

педагогический

университет», 2012

©Слинкина  М.В., Харина Г.В., 2012


Содержание

[1]
1. МЕТОДИЧЕСКИЕ УКАЗАНИЯ
ПО ОСНОВНЫМ РАЗДЕЛАМ КУРСА ХИМИИ

[2] 1.1.  Электронное строение атома

[2.1] Примеры решения задач

[3] 1.2. Периодический закон  и периодическая система  Д.И. Менделеева

[3.0.1] Относительная  электроотрицательность  элементов

[3.1] Примеры решения задач

[4] 1.3. Химическая связь

[4.1] Примеры решения задач

[5] 1.4. Классы неорганических соединений

[5.1] Примеры  решения задач

[6] 1.5. Элементы химической термодинамики и термохимии

[6.1] Примеры решения задач

[7] 1.6. Химическая кинетика и химическое равновесие

[7.1] Примеры решения задач

[8] 1.7. Электролитическая диссоциация. Реакции  ионного обмена

[8.1] Примеры решения задач

[9] 1.8. Растворы. Способы выражения концентрации растворов

[9.1] Примеры решения задач

[9.1.0.1] Р е ш е н и е. 1) Записываем выражение для молярной концентрации раствора NaOH :

[9.1.0.2] Р е ш е н и е. 1) Вспомним, что молярная доля растворенного вещества равна:

[9.1.0.3] Р е ш е н и е. 1)  Найдем массу 100 мл 15% раствора H2SO4:

[10] 1.9. Коллоидные растворы

[11] Примеры решения задач

[12] 1.10. Растворы неэлектролитов

[13] Примеры решения задач.

[14] 1.11.  Окислительно-восстановительные реакции

[15] Примеры решения задач

[16] 1.12. Электрохимические процессы в гетерогенных системах.

[17] Гальванические элементы

[17.1] Примеры решения задач

[18] 1.13. Коррозия металлов

[19] Примеры решения задач

[20] 1.14. Электролиз

[21] Примеры решения задач

[22] 1.15. Свойства и получение полимеров

[23] 2.  КОНТРОЛЬНЫЕ ЗАДАНИЯ

[24] 3. ВАРИАНТЫ КОНТРОЛЬНЫХ ЗАДАНИЙ

[25] ЛИТЕРАТУРА


Введение

Самостоятельная работа студента над курсом химии предусматривает изучение программного теоретического материала по лекциям, учебникам и учебным пособиям, выполнение индивидуальной контрольной работы, подготовку к лабораторному практикуму и экзамену (зачету).

Контрольная работа по курсу химии выполняется по индивидуальному варианту, включающему 10 задач для студентов заочной формы обучения и
20 задач для студентов дневной формы обучения. К выполнению контрольной работы следует приступать только после изучения и усвоения теоретической части курса. Изучать курс химии рекомендуется по отдельным темам, причем пока не усвоена предыдущая тема, не следует переходить к изучению последующей. Далее следует разобраться с типовыми задачами по изучаемой теме, решение которых приведено в конце каждого подраздела методических указаний. Если эти задачи не вызывают у Вас затруднений, то тогда можно смело обратиться к решению задачи, предложенной в индивидуальной контрольной работе.

Обратите внимание на решение расчетных задач: оно обязательно должно включать в себя уравнения химических реакций, математические выражения законов (или принципов), которые используются для расчетов, физический смысл всех величин, входящих в эти выражения, и числовые значения используемых констант. При решении задач необходимо поэтапно приводить все математические преобразования и только потом уже давать окончательный числовой ответ.

Контрольную работу следует выполнить в отдельной тетради в 12 листов. На титульном листе необходимо указать номер варианта, который для студентов заочной формы обучения соответствует двум последним цифрам номера студенческого билета или зачетной книжки. Студентам дневной формы обучения вариант контрольной работы выдается преподавателем, который и устанавливает требования к ее выполнению. Студенты всех форм обучения при оформлении работы сначала должны записать номер задачи и ее полное условие и только после этого изложить подробный ход решения.

Контрольная работа студентов заочной формы обучения должна быть аккуратно оформлена, датирована, подписана студентом и представлена в университет на рецензирование не позднее, чем за две недели до начала сессии. Контрольная работа, выполненная с ошибками, возвращается студенту. Ее следует доработать с учетом всех замечаний, сделанных преподавателем. Все необходимые исправления следует выполнять только в конце работы под заголовком  «Работа над ошибками», исправления в тексте не допускаются.

Контрольная работа, выполненная студентом по другому варианту, на рецензирование не принимается.


1. МЕТОДИЧЕСКИЕ УКАЗАНИЯ
ПО ОСНОВНЫМ РАЗДЕЛАМ КУРСА ХИМИИ

1.1.  Электронное строение атома

           Согласно представлениям квантовой механики электрон имеет двойственную природу: он ведет себя и как частица, и как волна. Электрон в атоме не имеет траектории движения. Квантовая механика рассматривает вероятность нахождения электрона в пространстве вокруг ядра.

Пространство вокруг ядра, в котором наиболее вероятно нахождение электрона, называется атомной орбиталью (АО).

Состояние электронов в атоме определяется энергией взаимодействия электронов с ядром. Эта энергия квантована, т.е. ее величина не может быть любой, а принимает лишь определенные значения, зависящие от некоторых величин n, l, ml, которые называются квантовыми числами. Поэтому атомная орбиталь – это энергетическое состояние электрона, для которого определены значения n, l и ml..

Главное квантовое число (n) характеризует уровень энергии электронов (энергетический уровень): чем больше значение n, тем больше энергия соответствующего уровня и средний размер электронного облака. Главное квантовое число принимает целочисленные значения:  n = 1, 2, 3 …

Своими значениями главное квантовое число  нумерует энергетические уровни, на которых могут находиться электроны в атоме. Число заполняемых электронами энергетических уровней в атоме численно равно номеру периода, в котором находится элемент: у атомов элементов первого периода – один энергетический уровень,  второго периода – два и т.д.  Каждый энергетический уровень (кроме первого) расщепляется на несколько энергетических подуровней. Эти подуровни энергий определяются орбитальным квантовым числом (l), которое характеризует также форму атомной орбитали. Орбитальное квантовое число принимает значения от 0 до (n – 1):         l = 0, 1, 2, 3 …   (n – 1).

В зависимости от величины l подуровни энергий различаются по типам, которые обозначаются латинскими буквами. Величине l = 0 соответствует s – подуровень, 1 – p, 2 – d, 3 – f. Чем больше значение l, тем выше энергия соответствующего подуровня в пределах одного и того же энергетического уровня.

1-й уровень (n=1,  l = 0) имеет    s – подуровень (1s); 2-й уровень (n=2,  l = 0,
l = 1) имеет s–  и  p– подуровни (2s 2p); 3-й уровень (n=3,  l = 0,  l = 1, l = 2) имеет
s–, p– и  d–подуровни  (3s3p3d) и т.д.

Магнитное квантовое число (ml) характеризует пространственную ориентацию  атомной орбитали. Его значения зависят от величины орбитального квантового числа: ml = – l … 0 … + l.  Например, для l = 1 (р – подуровень),
ml  = -1, 0, 1. Число значений, принимаемых ml, определяет число АО на данном подуровне. То есть на р-подуровне имеется 3АО , которым соответствуют три различных ориентации в пространстве, на s – подуровне  (l = 0,  ml  = 0)    1АО,  на  d  (l =2,  ml  = -2,-1, 0, 1, 2)   – 5АО и на    f  (l =3,   ml  = -3 -2,-1, 0, 1, 2, 3)   – 7АО.

Для условного изображения АО принят символ квадрата называемый квантовой или электронной ячейкой.

Электрон имеет собственный магнитный и механический моменты, которые объединили общим названием «спин», и  в связи с этим ввели четвертое квантовое число  ms   −  спиновое число, принимающее всего два значения: + ½ (↑) и – ½ (↓).

Порядок заполнения электронами энергетических уровней и подуровней подчиняется следующим правилам.

Принцип минимума энергии заключается в том, что заполнение электронами энергетических подуровней происходит в порядке возрастания их энергии. Так как энергия электронов на подуровнях главным образом определяется квантовыми числами n и l, то в первую очередь электроны заполняют подуровень, характеризующийся наименьшей суммой (n + l). Если для двух энергетических подуровней (n + l) одинакова, то прежде всего заполняется подуровень с меньшим значением n. Эти утверждения выражает правило Клечковского, с учетом которого последовательность заполнения электронами энергетических подуровней может быть представлена в виде следующего ряда: 1s < 2s < 2p < 3s < 3p <  4s < 3d  < 4p < 5s < 4d < 5p < 6s < 5d  ≈ 4f < 6p < 7s < 6d  ≈ 5f …

Принцип Паули определяет максимальное число электронов на атомной орбитали, которое не может быть больше двух: в атоме не может быть двух электронов с одинаковыми значениями всех четырех квантовых чисел. Поэтому если на атомной орбитали появляется второй электрон, то он будет иметь спиновое квантовое число противоположного знака. Принцип Паули позволяет определить максимальное число электронов (ē) на каждом энергетическом подуровне:
 s – подуровень – 2 ē  (s2);  p – подуровень – 6 ē  (p6);  d – подуровень – 10 ē  (d10);
f – подуровень – 14 ē  (f14).

Правило Гунда определяет порядок заполнения атомных орбиталей в пределах данного энергетического подуровня: атомные орбитали заполняются так, чтобы суммарное спиновое квантовое число электронов на подуровне было максимальным. Например, заселение вакантных d-АО пятью электронами возможно в только одним способом, отвечающим наименьшей энергии основного состояния  d5:

Распределение электронов по различным АО называется электронной конфигурацией (электронной формулой) атома. Например, электронная конфигурация атома кислорода 1s2 2s2 2p4 , а атома  натрия –  1s2 2s2 2p6 3s1.

          В электронной конфигурации энергетические уровни обозначаются цифрами 1, 2, 3.  Каждому энергетическому уровню соответствует определенное квантовое число  n = 1, 2, 3 … Энергетические подуровни обозначаются буквенными символами s, p, d, f . Каждый подуровень имеет соответствующее значение орбитального квантового числа  l: s – 0,  p – 1,  d – 2,  f – 3. Число электронов, находящихся на подуровне, изображается верхним индексом у буквенного символа, например, 1s2.

 При составлении электронной конфигурации необходимо пользоваться  Периодической системой Д.И. Менделеева, которая отражает электронное строение атома элемента (см. примеры решения задач).

Строение внешнего энергетического уровня, определяет химические свойства атома – способность принимать или отдавать электроны. Вступая в химическое взаимодействие, атом стремится приобрести наиболее устойчивую конфигурацию внешнего уровня – конфигурацию  ближайшего к нему инертного газа: двухэлектронную – ns2 (типа He) или восьмиэлектронную – ns2np6 (любого другого газа). Атомы, которые отдают  свои электроны другим атомам при химическом взаимодействии, превращаясь в положительно заряженные ионы, проявляют металлические или восстановительные свойства. Атомы, которые принимают электроны, превращаясь в отрицательно заряженные ионы, проявляют неметаллические или окислительные свойства. Заряд образующегося иона называется степенью окисления. В Периодической системе Д.И. Менделеева все элементы делятся на металлы, неметаллы и химически инертные благородные газы (8 группа, главная подгруппа). К металлам относятся sэлементы (элементы, у которых последним заполняется  s – подуровень внешнего уровня), кроме водорода и гелия; все d - и f – элементы (у них последними заполняются d – подуровень второго снаружи уровня и f – подуровень третьего снаружи уровня, соответственно); а также некоторые pэлементы (у них последним заполняется  p – подуровень внешнего уровня). Среди p – элементов металлы отделены от неметаллов диагональю, проходящей от B к At, и лежат ниже этой диагонали.

Примеры решения задач

Пример 1.  Какой  из подуровней: 4d  или  5s заполняется электронами в первую очередь?

Р е ш е н и е . Последовательность заполнения электронами подуровней в атоме определяется правилом Клечковского, которое предполагает сравнение значений суммы  (n + l) для каждого из подуровней. Следовательно, надо определить сумму квантовых чисел  n и  l для данных подуровней:  для 4d – подуровня n = 4,  l = 2,  n + l = 6;   для 5s – подуровня n = 5,  l = 0,  n + l = 5. В первую очередь будет заполняться  5s – подуровень,   так как для него значение  (n + l) меньше, чем для  4d– подуровня, то есть 5s – подуровень имеет меньшее значение энергии, чем 4d– подуровень, а заполнение подуровней электронами происходит в порядке возрастания их энергии.

Пример 2. Запишите электронную конфигурацию и электронную схему строения внешнего уровня атома элемента с зарядом ядра, равным +33. Определите, какими химическими свойствами обладает атом этого элемента.

Р е ш е н и е. 1) Определим, атом какого элемента имеет  Z = +33. Так как заряд ядра атома равен порядковому номеру N элемента в Периодической системе, то элементом  с  N = 33 является мышьяк (As).

2) Запишем электронную конфигурацию атома As (рассмотрим распределение электронов по энергетическим уровням и подуровням). Для этого определим координаты данного элемента в Периодической системе, т.е. номер периода (арабская цифра) и номер группы (римская цифра). Группы делятся на две подгруппы – главную (обозначают символом «А») и побочную (обозначают символом «В»). Номер периода равен числу энергетических уровней в атоме, а номер группы – числу электронов на внешнем уровне (валентных электронов). Координаты As: (4, VА) т.е. элемент расположен в четвертом периоде  (атом имеет четыре энергетических уровня); в пятой группе (имеет пять электронов на внешнем уровне) и главной подгруппе (р – элемент).

Число электронов в атоме равно заряду его ядра, следовательно, электронная оболочка  As содержит 33 электрона. Распределение  электронов по энергетическим уровням и подуровням проводим в соответствии с порядком их заполнения, учитывая максимальное число электронов на каждом подуровне: 1s2 2s2 2p6 3s2 3p6 3d10 4s2 4p3 

В электронной конфигурации выделим строение внешнего уровня, на котором находятся валентные электроны, способные участвовать в химическом взаимодействии – 4s2 4p3.

3) Изобразим электронную схему строения внешнего уровня, которая характеризует распределение электронов по энергетическим уровням, подуровням и атомным орбиталям, руководствуясь принципом Паули и правилом Гунда:

                                                         4p

↑↓

4s

4) Определим химические свойства атома  As.

Химические свойства атома определяются строением внешнего энергетического уровня. Вступая в химическое взаимодействие, любой атом стремится завершить внешний уровень. Атом As имеет 5 валентных электронов, поэтому завершение внешнего уровня возможно за счет присоединения трех электронов. Принимая их, атом мышьяка проявляет окислительные свойства:

As0  +  3ē   =   As3-     

         4s2 4p3                    4s2 4p6

Атом мышьяка может проявлять восстановительные свойства, отдавая электроны внешнего уровня - три   или все пять:

As0  -  3ē   =   As3+                                                        As0  -  5ē   =   As5+

         4s2 4p3                      4s2                                         4s2 4p3                      3s2 3p6 3d10

Пример 3.  Электронная конфигурация атома имеет вид: [Kr] 4d2 5s2. Определите, какой это элемент, и какие химические свойства проявляет атом  этого элемента.

Р е ш е н и е. 1) Определим координаты атома данного элемента в Периодической системе Д.И. Менделеева. Из электронной конфигурации атома видно, что главное квантовое число внешнего энергетического уровня равно пяти (n = 5), т.е. атом имеет пять энергетических уровней, следовательно, элемент расположен в 5-м периоде. Число валентных электронов равно четырем, значит, элемент расположен в IV группе. Так как незавершенным является d– подуровень (т.е. он заполняется  последним), то мы имеем дело с d- элементом, а все d- элементы расположены в Периодической системе Д.И. Менделеева в побочных подгруппах. Таким образом, элемент (Э) имеет следующие координаты: Э (5, IVВ). Как видно из Периодической системы, этот элемент – цирконий (Zr).

2) Установим химические свойства атома циркония.

Вспомним, что все d – элементы являются металлами. Значит, атом циркония проявляет восстановительные свойства и способен только отдавать свои валентные электроны, превращаясь в положительно заряженный ион со степенью окисления +4:
                      
Zr0  –    4ē    =   Zr4+

1.2. Периодический закон  и периодическая система  Д.И. Менделеева

Современная формулировка Периодического закона: свойства простых веществ, а также формы и свойства соединений элементов находятся в периодической зависимости от величины зарядов ядер их атомов.

Физический смысл Периодического закона состоит в том, что с возрастанием заряда ядра происходит периодическое повторение сходного строения внешнего энергетического уровня атомов элементов. В соответствии с этим физические и химические свойства атомов элементов периодически повторяются.

Периодическая система является графическим выражением Периодического закона. Все элементы в Периодической системе расположены в виде горизонтальных и вертикальных рядов, называемых периодами и группами.

Период – это горизонтальная последовательность элементов, в атомах которых происходит заполнение электронами одинакового числа энергетических уровней. Номер периода определяет число энергетических уровней в атомах элементов данного периода и соответствует значению главного квантового числа внешнего энергетического уровня

Группа - это вертикальная последовательность  химических элементов.  Номер группы указывает на число валентных электронов, т.е. тех, которые могут участвовать в образовании химической связи. В одну группу объединяются элементы с одинаковым числом валентных электронов независимо от их электронного типа (s-, p-, d-, f-). Номер группы совпадает с высшей валентностью элемента в возбужденном состоянии и отвечает высшей положительной степени окисления атомов (кроме F, O и Br).

Каждая группа состоит из двух подгрупп – главной и побочной. В главную подгруппу входят s- и р- элементы, а в побочную – d- элементы.  То есть в каждой подгруппе объединены элементы, атомы которых имеют сходное строение валентного уровня. Такие элементы  называют электронными аналогами.

Важнейшие характеристики атома, которые изменяются периодически от величины заряда ядра и в конечном итоге определяют химические свойства элементов и их соединений, – это  радиус атома, энергия  ионизации, энергия сродства к электрону и электроотрицательность.

Эффективный радиус атома (rат) принимают равным половине межъядерного расстояния в молекулах или кристаллах соответствующих простых веществ. В пределах одного периода (при движении слева направо) при неизменном числе энергетических уровней  заряд ядра атома увеличивается. Это приводит к возрастанию силы электростатического притяжения валентных электронов к ядру, вследствие чего происходит сжатие орбиталей, т.е. атомный радиус уменьшается. Внутри группы (при движении сверху вниз) заряд ядра атома и число энергетических уровней возрастают. Вследствие проявления эффекта экранирования (защиты валентных электронов от влияния ядра атома электронами внутренних энергетических уровней) силы электростатического притяжения между ядром и валентными электронами уменьшаются, и радиус атома увеличивается.

Энергия ионизации (Еи) – это энергия, необходимая для отрыва одного электрона от невозбужденного атома. Еи  является количественной характеристикой восстановительных свойств атомов. Чем меньше величина Еи, тем сильнее восстановительные свойства атома.

Энергия сродства к электрону (Ее) – это энергия, которая выделяется при присоединении электрона  к нейтральному атому. Ее характеризует окислительные свойства атомов. С увеличением энергии сродства к электрону окислительная способность атома повышается.

Электроотрицательность (ЭО) – это способность атома в молекуле притягивать к себе  чужие электроны, участвующие в образовании химической связи.  ЭО = (Еи   +  Ее) / 2.

В настоящее время используется шкала относительных электроотрицательностей,  в которой ЭО атома фтора, как самого сильного окислителя, условно принята равной 4 (табл.1). При образовании молекулы электроны смещаются от атома с меньшей ЭО к атому  с большей ЭО. Внутри периодов наблюдается общая тенденция роста ЭО атомов, а в группах – ее падение.

Химические свойства атома зависят от конфигурации внешнего энергетического уровня, rат,  Еи,  и  Ее. В пределах периода (слева направо) rат уменьшается, Еи, и  Ее повышаются. В результате способность атомов к отдаче электрона уменьшается, а к присоединению электрона увеличивается. Таким образом, в периоде металлические свойства атомов элементов ослабляются, а неметаллические – усиливаются. В главной подгруппе (сверху вниз) rат  увеличивается, а Еи уменьшается, в результате способность атомов отдавать свои электроны повышается, а способность принимать чужие электроны снижается. Таким образом, в главной подгруппе металлические свойства атомов элементов усиливаются, а неметаллические ослабевают.

 В периоде с ростом степени окисления основные свойства гидроксидов ослабевают, а кислотные свойства усиливаются. В подгруппах (сверху вниз) кислотные свойств кислородсодержащих соединений ослабевают, а основные свойства увеличиваются. Так, La(OH)3 значительно более сильное основание, чем Al(OH)3;  H3AsO3  более слабая кислота, чем  HNO3.

Таблица 1

Относительная  электроотрицательность  элементов

Периоды

Группы

I

II

III

IV

V

VI

VII

1

H

2,1

2

Li

1,0

Be

1,5

B

2,0

C

2,5

N

3,0

O

3,5

F

4,0

3

Na

0,9

Mg

1,2

Al

1,6

Si

1,8

P

2,1

S

2,5

Cl

3,0

4

K

0,8

Ca

1,0

Ga

1,6

Ge

1,8

As

2,0

Se

2,4

Br

2,8

5

Rb

0,8

Sr

1,0

In

1,7

Sn

1,8

Sb

1,9

Te

2,1

I

2,5

6

Cs

0,7

Ba

0,9

Tl

1,8

Pb

1,6

Bi

1,9

Po

2,0

At

2,2

Примеры решения задач

Пример 1. Объясните, почему алюминий и скандий находятся в одной группе, но в разных подгруппах?

Р е ш е н и е. 1) Запишем электронные конфигурации атомов и выделим валентные уровни:  Al    1s2 2s2 2p6  3s2 3p1             

                     Sc    1s2 2s2 2p6 3s2 3p6 4s2 3d1

2) Обоснуем расположение элементов Al и Sc в одной группе, но в разных подгруппах. Атомы алюминия и скандия  имеют одинаковое число валентных электронов – три.  Следовательно, Al и Sc  – это элементы одной группы  (III). Однако характер заполнения валентного уровня у этих атомов различен. Алюминий – это p – элемент, у него последним заполняется p – подуровень внешнего энергетического уровня, поэтому валентными являются электроны 3s23p1. Скандий – это d- элемент, у которого в последнюю очередь заполняется d– подуровень предпоследнего энергетического уровня, поэтому валентные электроны – 4s23d1. Именно это является причиной расположения атомов Al и  Sc  в разных подгруппах: Al (IIIА) – в главной, а Sc (IIIB) – в побочной подгруппе.

Пример 2. Руководствуясь положением элементов в Периодической системе, определите, какой из атомов – сера или теллур проявляет более сильные неметаллические свойства.

Р е ш е н и е. 1) Определяем координаты этих элементов в Периодической системе: S (3, VIA) и Те (5, VIA), т.е. эти элементы являются электронными аналогами, так как расположены в одной (главной) подгруппе VI группы.

2) Составляем электронные формулы атомов этих элементов и выделяем строение внешних уровней (именно они ответственны за химические свойства любого атома):                    
S – 1s2 2s2 2p6 3s2 3p4   ,  Те – 1s2 2s2 3s2 Зр6 3d10 4s26 4d10 5s24 

Действительно, атомы S и Те имеют сходное строение внешнего уровня, который можно представить в виде ns2nр4 , т.е. на внешнем уровне находится 6 валентных электронов.

3) Сравним неметаллические свойства атомов S и Те. Неметаллические свойства определяются способностью атома присоединять электроны при их химическом взаимодействии. Неметаллические свойства атомов зависят от конфигурации внешнего уровня, радиуса атома (гат) и величины энергии сродства к электрону (Ее).

Как уже отмечалось, элементы S и Те расположены в одной группе, имеют сходное строение внешнего уровня - ns2nр4. Однако атом S имеет три энергетических уровня, а атом Те – пять, поэтому валентные электроны у S расположены ближе к ядру.  Радиус атома S меньше, чем радиус атома Те, а энергия сродства к электрону больше, чем Ее  атома Te (в главной подгруппе сверху вниз гат увеличивается, а Ее  уменьшается). Поэтому атом S обладает большей способностью присоединять электроны. Следовательно, атом S по сравнению с атомом Те проявляет более  сильные неметаллические свойства.

Пример 3. Руководствуясь Периодической системой, определите какой из элементов – магний или алюминий обладает более выраженными  металлическими свойствами.

Р е ш е н и е. 1) Химические свойства элементов определяются электронным строением внешних уровней их атомов. Запишем электронные конфигурации атомов магния и алюминия. Они расположены в третьем периоде (имеют одинаковое число энергетических уровней, равное трем). Магний – элемент второй группы, имеет два валентных ē. Алюминий – элемент третьей группы, имеет три валентных ē. Оба элемента расположены в главных подгруппах, т.е. все валентные электроны находятся на внешнем уровне. Отсюда электронные конфигурации внешних уровней: Mg  2s2,  Al  3s23p1.

2) Сравним металлические свойства атомов этих элементов – способность отдавать электроны при химическом взаимодействии. Металлические свойства зависят от конфигурации внешнего уровня, радиуса атома (rат) и энергии ионизации (Еи). Магний и алюминий находятся в одном периоде. При переходе от Mg к Al происходит увеличение заряда ядра и числа ē на внешнем уровне, которые все сильнее удерживаются ядром атома вследствие уменьшения rат. При этом Еи возрастает и способность атома к отдаче электронов уменьшается. Следовательно, магний обладает более сильными металлическими свойствами, чем алюминий.

1.3. Химическая связь

Выделяют три типа химической связи: ковалентную, ионную и металлическую.

Ковалентная связьхимическая связь, осуществляемая общими электронными парами. В соответствии с методом валентных связей (ВС) ковалентная связь между двумя атомами осуществляется общей для этих атомов парой электронов с противоположными спинами. В момент образования связи атомные орбитали перекрываются, что приводит к увеличению электронной плотности между  ядрами  взаимодействующих атомов и к взаимному притяжению ядер к области повышенной электронной плотности. В результате этого происходит выделение энергии и потенциальная энергия системы уменьшается.

Общая для двух атомов электронная пара может образовываться по двум механизмам: обменному или донорно-акцепторному.

При обменном механизме два связываемых атома (А и В) предоставляют для образования связи по одному неспаренному электрону, как бы обмениваясь ими:

↑↓

                                             А                            +  В              →     А   В А

Донорно-акцепторный механизм образования связи заключается в том, что один атом А (донор) на образование связи предоставляет пару электронов, а другой атом В (акцептор) – вакантную  атомную орбиталь.

Различают две разновидности ковалентной связи: неполярную и полярную.

Ковалентная неполярная связь – это связь, при которой область повышенной электронной плотности расположена симметрично относительно ядер обоих атомов. Такая связь образуется между атомами с одинаковой электроотрицательностью (ЭО), например, в молекулах Cl2, O2, H2  и др.

Ковалентная полярная связь – это  связь, при которой область повышенной электронной плотности смещена к ядру атома с большей ЭО. В результате этот атом приобретает эффективный отрицательный заряд, а на другом менее электроотрицательном атоме возникает равный по величине эффективный положительный заряд. Такая система представляет собой электрический диполь. Полярная связь образуется между атомами с разной ЭО, например, в молекулах HCl, HI,  H2O, H2S, CO и др. Чем больше разность электроотрицательностей атомов, образующих связь (∆ЭОА – В), тем выше полярность связи.

Важнейшие свойства ковалентной связи – насыщаемость и направленность. Насыщаемость – это способность атомов образовывать ограниченное число ковалентных связей. В случае обменного механизма число связей равно числу неспаренных валентных электронов атома. Способность атома к образованию химических связей характеризуется валентностью.

Валентность определяется как число химических связей,  которыми данный атом  соединен с другими атомами. Она зависит от того, в каком состоянии - основном или возбужденном находится атом. Основное состояние – это устойчивое состояние с наименьшей энергией. При возбуждении спаренные валентные электроны разъединяются и переходят с одного подуровня на свободные АО другого, энергетически более высокого подуровня в пределах внешнего энергетического уровня. В результате число неспаренных электронов увеличивается, и атом данного элемента образует максимально возможное для него число химических связей, проявляя при этом высшую валентность и высшую положительную степень окисления, равную номеру группы в Периодической системе (см. примеры решения задач).

Ковалентная связь имеет направленность, которая обусловливает пространственную структуру молекулы, т.е. ее геометрическую форму. В зависимости от способа перекрывания АО различают  σ (сигма)- ,  π (пи)- и  
δ (дельта)-   связи.

σ – связь обладает осевой симметрией относительно межъядерной оси, и область перекрывания АО лежит на межъядерной оси. Ее могут образовывать s – АО, p – АО и d – АО. Именно σ – связи определяют пространственную конфигурацию молекул:

π – связь образуется при перекрывании АО, расположенных параллельно друг другу. Область перекрывания лежит по обе стороны от межъядерной оси. В образовании π – связи могут участвовать p –  и d – АО:

δ – связи образуют только d – АО.

Сигма – связь является самой прочной связью и всегда образуется в первую очередь. Между двумя атомами в молекуле возможна лишь одна σ – связь.

Для объяснения геометрической структуры молекул (или направленности ковалентной связи) используют представление о гибридизации атомных орбиталей центрального атома в молекуле АВn.

Гибридизация это выравнивание энергии различных АО у атома А в результате их смешения перед химическим взаимодействием, что приводит к образованию гибридных орбиталей. В гибридизации участвуют только АО одного уровня, например, 2s и 2p. Каждому виду гибридизации АО соответствует определенная геометрическая форма молекулы. Например, sp– гибридизации (две связи) соответствует линейная форма молекулы (BeCl2), sp2– гибридизации (три связи) – плоская треугольная (BCl3), sp3 - гибридизации (четыре связи) – тетраэдрическая (CH4).

Ионная связь – связь между ионами, осуществляемая их электростатическим взаимодействием. Ионная связь возникает между атомами металлов и неметаллов, резко отличающимися по своей электроотрицательности. Механизм образования ионной связи заключается в переходе электронов от одного атома к другому (более электроотрицательному), в результате чего атомы превращаются в противоположно заряженные ионы (катион и анион) и происходит их электростатическое взаимодействие.  Свойства ионной связи – ненаправленность и ненасыщаемость.

Металлическая связь – это связь, образованная в результате перекрывания валентных орбиталей атомов металлов, в результате чего электроны свободно перемещаются из одной орбитали в другую, осуществляя связь между всеми атомами кристалла металла.

Примеры решения задач

Пример 1. Объясните механизм образования ковалентной химической связи в молекуле HBr и оцените степень ее полярности.

Р е ш е н и е. 1) Для объяснения механизма образования ковалентной химической связи необходимо определить, какие электроны участвуют в образовании этой связи. Запишем электронные конфигурации атомов и электронные схемы строения их валентных уровней; изобразим форму АО, участвующих в образовании связи.

       Н           1s1                           

Z = +1

      Br           1s22s22p63s23p63d104s24p5

Z = +35                                                   4p   

↑↓

↑↓

                                                   4s

↑↓

Для образования ковалентной связи атомы водорода и брома предоставляют по одному неспаренному электрону с антипараллельными спинами: атом  Н – электрон, находящийся на s – АО (форма АО – сфера), а атом Br – электрон с
p – АО (форма АО – гантель).

2) Покажем механизм образования ковалентной связи в молекуле HBr.

В молекуле HBr связь создается за счет перекрывания двух атомных орбиталей: s – АО и p – АО  с образованием между ядрами атомов H  и Br зоны повышенной электронной плотности:

                                                         H    Br

3) Для определения степени полярности связи рассчитаем разность электроотрицательностей атомов, образующих молекулу: ЭОН = 2,2; ЭОBr =  2,8; т.е. ЭОHBr = 0,6, поэтому связь в молекуле HBr ковалентная полярная.

4)  Определим вид химической связи в зависимости от способа перекрывания АО взаимодействующих атомов. В молекулах с одинарной химической связью (а именно таковой является молекула HBr) всегда образуется σ – связь как более прочная. В случае σ – связи область перекрывания АО расположена на линии, соединяющей ядра двух атомов.

Пример 2. Определите химические свойства, валентность и возможные степени окисления атома углерода   в основном и возбужденном состояниях.

Р е ш е н и е. 1) Рассмотрим основное состояние атома углерода. Так как химические свойства атома определяются его электронным строением, составим электронную конфигурацию атома  С  и выделим строение валентного уровня:

                С     Z = +6  ,             1s22s22p2 

2) Составим электронную схему валентного уровня и определим  химические свойства атома  С, его валентность и степень окисления.

                                             

↑↓

       2s                    2p

Валентность атома определяется числом неспаренных электронов валентного уровня. Из данной схемы видно, что атом углерода имеет два неспаренных валентных электрона, значит в основном состоянии  валентность атома углерода равна двум (В=II), т.е. атом углерода может образовывать две химические связи. Вступая во взаимодействие с другими атомами, атом С стремится завершить свой внешний уровень. Поэтому он может отдать эти два неспаренных электрона, проявляя при этом восстановительные свойства и превращаясь в положительно заряженный ион со степенью окисления +2:  С0        −       2 ē     =   С+2

2s22p2    2s2 (типа He)

Но атом углерода, как неметалл, может принимать недостающие до завершения внешнего уровня  четыре электрона, проявляя окислительные свойства и превращаясь в отрицательно заряженный ион со степенью окисления  – 4:

С0   +  4 ē   =   С-4

2s22p2  2s22p6 (типа Ne)

3) Рассмотрим возбужденное состояние атома углерода. Для возбуждения атома необходимо наличие свободной АО внутри валентного уровня и спаренных электронов. Из электронной схемы строения внешнего уровня атома углерода видно, что атом С имеет вакантную АО на 2p – подуровне, а из четырех валентных электронов два электрона (2s2) спарены. Следовательно, атом углерода может находиться в возбужденном состоянии. При возбуждении происходит распаривание 2s2- электронов и переход их с 2s- на 2p – подуровень:

С0…2s22p2                                                →  С* …  2s12p3                  

↑↓

                                 

    2s           2p               2s           2p

Основное состояние                             Возбужденное состояние

При возбуждении число неспаренных электронов увеличивается до четырех. Значит в возбужденном состоянии атом углерода проявляет валентность В=IV и образует четыре химические связи. В возбужденном состоянии атом С может только отдать на связь свои 4ē, проявляя восстановительные свойства и превращаясь в положительно заряженный ион со степенью окисления +4:

С0      –      4 ē     =   С+4

          2s22p2                        1s2 (типа He)

Пример 3. Определите, какая связь CN  или CH является  более полярной. Укажите, к ядру какого атома происходит смещение общей электронной пары.

Р е ш е н и е.  Для определения полярности связи необходимо найти разность электроотрицательностей атомов (∆ЭО), образующих эти связи. Из табл.1 выписываем значения ЭО этих атомов и находим ∆ЭО:
ЭОС =2,5;  ЭОN = 3,0;    ЭОH = 2,1;  ∆ЭОCN =3,0 – 2,5 =  0,5; ∆ЭОCH = 2,5 – 2,1 = 0,4.

Известно, что чем больше ∆ЭО  атомов, образующих связь, тем выше полярность связи. Поэтому более полярной является связь CN. При образовании ковалентной связи общая электронная пара смещается к ядру атома с большей ЭО. В химической связи CN общая электронная пара смещена к атому N, а в химической связи CH  – к атому С.

1.4. Классы неорганических соединений

 Основные классы неорганических соединений - оксиды, гидроксиды (кислоты и основания) и соли. Между классами неорганических соединений существует взаимосвязь: из веществ одного класса можно получить вещества другого класса. Такую связь называют генетической.

Генетическую связь между классами неорганических соединений, т.е. возможность их взаимных переходов, можно выразить схемой:

Металл  →  Основной оксид      →      Основание     →       Соль

Металл   →  Амфотерный оксид    →      Амфотерный  гидроксид  →   Соль

Неметалл    →   Кислотный оксид    →      Кислота   →     Соль

Рассмотрим наиболее сложные вопросы этой темы: свойство амфотерность и один из классов неорганических соединений – соли.

Амфотерность гидроксидов и оксидов – способность некоторых плохо растворимых гидроксидов или оксидов металлов проявлять кислотные или основные свойства в зависимости от природы партнера по реакции в кислотно-основном взаимодействии. Амфотерные оксиды и гидроксиды могут взаимодействовать как с кислотами (подобно основаниям), так и со щелочами (подобно кислотам), образуя в том и в другом случае соли.  В реакциях: Zn(OH)2 + H2SO4  =  ZnSO4   + 2H2O

   ZnO + H2SO4   =  ZnSO4  +   H2O

Zn(OH)2 и ZnO, реагируя с кислотой, проявляют основные свойства.  А в реакциях:

Zn(OH)2 +  2NaOH  =  Na2Zn02 + 2H2O

           (H2Zn02)                        цинкат натрия

ZnO    +    2NaOH  =  Na2Zn02 +   H2O

Zn(OH)2 и ZnO проявляют кислотные свойства.

К амфотерным гидроксидам относятся Ве(ОН)2, А1(ОН)3, Zn(OH)2, Sn(OH)2, Pb(OH)2, а также гидроксиды d- металлов в их промежуточных степенях окисления.

Соль это продукт взаимодействия основания и кислоты.  Все соли делят на три группы: средние (или нормальные), кислые и основные.

Средние (или нормальные) соли – это продукты полного замещения атомов водорода кислоты на атомы металла:    H2SO4 + 2NaOH = Na2SO4 + 2Н2О.

Na2SO4 – сульфат натрия – средняя соль, так как она получена в результате полного замещения ионов Н+ серной кислоты на катионы Na+.

Кислые соли – это продукты неполного замещения атомов водорода кислоты на атомы металла:    H2SO4 + NaOH = NaHSO4 + Н2О.

NaHSO4 – гидросульфат натрия (гидро - атом водорода) – кислая соль, так как в ее состав входит сложный анион HSO4 (гидросульфат), т.е. один ион водорода H2SO4  не замещен на ион Na+. Кислые соли образуются, если основания взято меньше, чем требуется для полной нейтрализации кислоты. Кислые соли образуют многоосновные кислоты (основность кислоты определяется числом атомов водорода). Одноосновные кислоты кислых солей не образуют. Кислые соли также могут быть получены при взаимодействии избытка кислоты со средними солями:   Na2SO4 + H2SO4 = 2NaHSO4 .

Основные соли – продукт неполного замещения гидроксильных групп основания кислотными остатками: Cu(OH)2 + HC1 = (СиОН)С1 + Н2О.

(CuOH)Cl – гидроксохлорид меди (гидроксо-группа ОН) – основная соль, так как содержит сложный катион (CuOH)+ с одной незамещенной группой ОН  в  Cu(ОН)2 на хлорид-ион Cl. Основные соли получаются, когда взятого количества кислоты недостаточно для образования средней соли. Основные соли образуют многокислотные основания (кислотность основания определяется числом ОН  – групп). Однокислотные основания основных солей не образуют. Основные соли получают при добавлении небольших количеств щелочей к растворам средних солей металлов, имеющих малорастворимые основания, например:

А1С13 + 2NaOH = [AI(OH)2] Cl + 2NaCl

 

Примеры  решения задач 

 Пример 1. Докажите, что оксид свинца (II) имеет амфотерный характер.

Р е ш е н и е.  Для доказательства амфотерного характера любого оксида (или гидроксида) необходимо привести уравнения химических реакций, в которых эти соединения проявляют основные и кислотные свойства.

1) Основные свойства оксида свинца (II) можно проиллюстрировать на примере взаимодействия РbО с веществами, имеющими кислотный характер, т.е. с кислотой и кислотным оксидом:  РbО + 2НNO3 =  Pb(NO3)2 + Н2О

 PbO  + SiO2   =   PbSiO3

В приведенных реакциях РЬО проявляет свойства основного оксида, так  как реагирует с кислотой и кислотным оксидом и образует соли, в состав которых свинец входит в виде катиона  Pb2 + .

2) Кислотные свойства оксида свинца (II) можно продемонстрировать с помощью реакций взаимодействия РbО со щелочами и с основными оксидами:

РbО + 2NaOH = Na2PbO2 +  Н2О   в расплаве

        РbО +  Na2O  =  Na2PbO2   в расплаве

РbО + 2NaOH +  Н2О =  Na2 [Pb(OH)4] – в растворе

В этих реакциях РbО выступает в роли кислотного оксида и образует соли, в состав которых свинец входит в виде аниона РbО22- или комплексного иона b(ОН)4]2-.

Таким образом, РbО является амфотерным оксидом, так как он проявляет и основные, и кислотные свойства.

Пример 2. Напишите уравнения всех возможных реакций между следующими веществами, взятыми попарно: оксид калия, оксид фосфора (V), гидроксид натрия, серная кислота, гидросульфат натрия, гидроксид бериллия.

Р е ш е н и е. 1) Устанавливаем принадлежность каждого из этих веществ к определенному классу неорганических соединений: К2О – основный оксид, Р2О5 – кислотный оксид, NaOH – основание (щелочь), H2SO4 – кислота, NaHSO4 – кислая соль, Ве(ОН)2 – амфотерный гидроксид.

2) Используя сведения о химических свойствах оксидов, гидроксидов и солей, напишем уравнения реакций между представителями этих классов соединений.

Основный оксид  К2О может взаимодействовать с кислотным оксидом, кислотой и амфотерным гидроксидом:    2О + 2Р2О5 = 2К3РО4

К2О  + H2SO4    =    K2SO4   + Н2О

К2О  + Ве(ОН)2   =    К2ВеО2 + Н2О

          (Н2ВеО2)

Кислотный оксид Р2О5 может взаимодействовать с основным оксидом, основанием и амфотерным гидроксидом:

Р2О5 + ЗК2О = 2К3РО4

Р2О5+  6NaOH =  2Na3PO4 +  3Н2О

Р2О5  +  ЗВе(ОН)2 =  Ве3(РО4)2 +  3Н2О

Основный гидроксид  NaOH реагирует с кислотным оксидом, кислотой, амфотерным гидроксидом и кислой солью:

6NaOH +  Р2О5   = 2Na3PO4 + ЗН2О

2NaOH + H2SO4 = Na2SO4 + 2Н2О

2NaOH + Ве(ОН)2  =   Na2BeО2   +   2H2O

             (H2BeО2)

NaOH  + NaHSO4  =    Na2SO4  +  H2O

Амфотерный гидроксид Ве(ОН)2 ( или H2BeО2) реагирует с основным оксидом, основанием, кислотным оксидом и кислотой :

H2BeО2 + К2О =  К2ВеО2 + Н2О

   H2BeО2  + 2NaOH  = Na2BeО2 + 2H2O

Амфотерный гидроксид Ве(ОН)2 при взаимодействии с основным оксидом и щелочью проявляет свойства кислоты H2BeО2:

3Ве(ОН)2 + Р2О5 = Ве3(РО4)2 + ЗН2О

Ве(ОН)2 +  H2SO4   = BeSO4  +  2Н2О

Амфотерный гидроксид Ве(ОН)2 при взаимодействии с кислотным оксидом и кислотой проявляет основные свойства.

Кислая соль NaHSO4  реагирует с основным оксидом и основанием:

2NaHSO4 + К2О = Na2SO4 + K2SO4 + Н2О

NaHSO4   + NaOH = Na2SO4 + H2O

Следовательно, из всех приведенных веществ попарно не взаимодействуют только К2О и NaOH, поскольку основные оксиды не вступают в реакции с основаниями.

Пример 3. Объясните закономерность в изменении кислотно-основных свойств гидроксидов элементов третьего периода Периодической системы
Д.И. Менделеева в их высших степенях окисления.

Р е ш е н и е. 1) Введем понятие «гидроксиды». Гидроксиды – это сложные вещества, в состав которых входит гидроксильная группа ОН. Условно класс гидроксидов можно описать с помощью общей формулы  Э−О−Н (Э – химический элемент).

2) Гидроксиды делят на три группы: основные, кислотные и амфотерные. Рассмотрим, как определяется  принадлежность гидроксидов к кислотам, основаниям или амфотерным гидроксидам. Принадлежность гидроксида к классу кислот или оснований определяется местом разрыва химических связей в Э−О−Н. Если разрывается связь О−Н (Э −О ↓−НН+ + ЭО-), то такой гидроксид относится к классу кислот, поскольку при разрыве связи образуется ион H+ – носитель кислотных свойств. Если разрывается связь Э −О (Э ↓−О−Н → Э+ + ОН), то гидроксид относится к классу оснований, так как образуется ион ОН – носитель основных свойств. Если же, в зависимости от среды, разрываются обе связи Э−О и О−Н, то такие гидроксиды проявляют двойственность свойств и называются амфотерными.

3) Место разрыва химической связи в гидроксиде Э−О−Н зависит от положения элемента в Периодической системе, что и определяет относительную прочность связи между  Э−О и О−Н. Силы притяжения между противоположно заряженными частицами тем значительнее, чем больше заряд каждой из них и меньше радиус.

4) Записываем формулы гидроксидов элементов третьего периода Периодической системы в их высших степенях окисления (высшая степень окисления атома элемента соответствует номеру группы):

  +    +2            +3           +4        +5       +6            +7

NaOH — Mg(OH)2 —   А1(ОН)3 —    H2SiО3 —   HNO3 —    H2SO4 —   НС1О4

5) Сравниваем относительную прочность связей Э−O и О−Н у высших гидроксидов третьего периода, учитывая, что при переходе от Na к CI наблюдается уменьшение радиуса атома. Благодаря своим малым размерам ион водорода Н+ в NaOH и Mg(OH)2 сильнее взаимодействует с кислородом, чем ион металла. Вследствие этого менее прочными оказываются связи Na−О и Mg−О, поэтому NaOH и Mg(OH)2 являются основаниями. В результате дальнейшего увеличения заряда и уменьшения радиуса атома при переходе к А1 связи А1−О и О−Н становятся близки по прочности, и А1(ОН)3 является типичным амфотерным гидроксидом. Наконец, у последних четырех соединений вследствие еще большего увеличения заряда и уменьшения радиуса атомов заметно увеличивается прочность связи Э−О и уменьшается прочность связи О−Н, поэтому гидроксиды H2SiO3, HNO3, H2SO4 и НСlO4 являются кислотами.

1.5. Элементы химической термодинамики и термохимии

Химическая термодинамика – это часть термодинамики, рассматривающая превращения энергии и работы при химических реакциях. Термохимия – раздел химической термодинамики в приложении к тепловым эффектам химических реакций.

Тепловой эффект химической реакции – это количество теплоты, которое выделяется или поглощается при реакции. При этом происходит изменении внутренней энергии системы (U)суммы кинетической и потенциальной энергий всех частиц, составляющих систему: U = U2U1 (U1 – начальное состояние системы, U2 – конечное состояние системы).

В соответствии с первым законом термодинамики (законом сохранения энергии) изменение внутренней энергии  закрытой системы определяется количеством теплоты Q, полученной системой из окружающей среды, и работой A, произведенной системой над окружающей средой:

                          ∆U = QA,                    (1)

где   A = PV – механическая работа расширения.

Для изобарных процессов (Р = const, V ≠ 0) в термодинамике вводится новая функция  энтальпия  H = U + PV и  Q определяется как изменение энтальпии
H = H2H1:

                             QP = ∆H,       (2)

где  H – тепловой эффект химической реакции при P = const.

Реакции, протекающие с выделением теплоты, называются экзотермическими. В экзотермических реакциях внутренняя энергия и энтальпия системы уменьшаются, H < 0. Реакции, протекающие с поглощением теплоты, называются эндотермическими. В эндотермических реакциях внутренняя энергия  и энтальпия системы увеличиваются, H > 0.

В основе термохимических расчетов лежит закон Гесса: тепловой эффект химической реакции зависит только от природы и физического состояния исходных веществ и продуктов реакции и не зависит от пути реакции, т.е. числа промежуточных стадий. Закон Гесса констатирует тот факт, что U и H являются функциями состояния системы, т.е. их изменение (U и H) зависит только от начального и конечного состояния системы.

Следствие из закона Гесса:

Тепловой эффект химической реакции равен разности между суммой энтальпий образования продуктов реакции и суммой энтальпий образования исходных веществ:

H0298   =     ∑ν ∆H0f, 298         –      ∑ν′ ∆H0f, 298 ,      (3)

продукты   исходные

 реакции              вещества

             (конечное состояние   (начальное состояние

      системы)   системы)

где   ν и  ν′ - стехиометрические коэффициенты в уравнении реакции;

H0f,298 – стандартная энтальпия образования вещества, кДж/моль.

Стандартная энтальпия образования вещества (H0f,298) – это тепловой эффект образования одного моля данного вещества из простых веществ в стандартных условиях (Т = 298К, Р = 1 атм.). Значения H0f, 298 приводятся в справочной литературе (см. табл. 2). Для простых веществ H0f, 298 = 0.

Наблюдения показывают, что самопроизвольно, т.е. без затраты работы извне, могут идти как экзотермические, так и эндотермические реакции, если последние сопровождаются увеличением  неупорядоченности системы (например, реакции, в которых из твердых веществ образуются газообразные вещества).

Степень неупорядоченности системы выражается термодинамической величиной – энтропией S. Чем выше неупорядоченность системы, тем больше ее энтропия.  S также, как  U  и  H, является функцией состояния системы.  Для вычисления изменения энтропии (S =S2S1) в химических реакциях  используют следствие из закона Гесса:

S0298   =     ∑ν ∙S0 298         -          ∑ν′ ∙ S0 298 ,      (4)

      продукты   исходные

    реакции   вещества

гдеS0298  – изменение энтропии реакции при стандартных условиях, Дж/моль;

ν  и  ν′  – стехиометрические коэффициенты в уравнении реакции;

S0 298   – стандартная энтропия вещества, Дж/моль∙К.

Известны абсолютные значения энтропии веществ, они приводятся в справочной литературе (см. табл. 2).

Термодинамическая величина, связанная с энтальпией и энтропией, называется энергией Гиббса (изобарно-изотермическим потенциалом) и обозначается буквой G:
G = HTS.  В изобарно-изотермических условиях

G = ∆H –  TS         (5)

Величина ∆G является критерием направления и предела самопроизвольного протекания химических реакций в закрытых системах при Р,Т = const:  G ≤ 0.

Если  G < 0, то реакция протекает самопроизвольно в прямом направлении (ему отвечает убыль энергии G2 < G1); если G > 0, то самопроизвольно протекает обратная реакция (G2 > G1); если G = 0, то система находится в состоянии равновесия, при котором  G = Gmin. Так как G является функцией состояния системы, то для стандартных условий изменение энергии Гиббса химической реакции G0298 рассчитывается следующим образом:

G0298 =  ∑ν ∙ ∆G0f, 298   –  ∑ν′ ∆G0f, 298 ,      (6)

 продукты      исходные

 реакции      вещества

где ν и  ν′ - стехиометрические коэффициенты в уравнении реакции;

G0f, 298   –  стандартная энергия Гиббса образования вещества, кДж/моль.

Стандартная энергия Гиббса образования вещества (∆G0f, 298) это изменение энергии Гиббса при реакции образования одного моля вещества из простых веществ при стандартных условиях. Значения  G0f,298  – это справочные данные, для простых веществ G0f, 298 = 0 (см. табл. 2). Для расчета G при температуре, отличающейся от стандартной (Т ≠ 298К), используется соотношение:

GТ   =  ∆H0298TS0298 ,       (7)

где H0298 – тепловой эффект химической реакции при стандартных условиях, энтальпийный фактор; S0298 – изменение энтропии химической реакции при стандартных условиях;  TS0298  энтропийный фактор.

Знак и величина G, а, следовательно, и возможность самопроизвольного протекания процесса при заданных температуре и давлении зависят от соотношения энтальпийного и энтропийного факторов.

Примеры решения задач

Пример 1. Рассчитайте тепловой эффект реакции образования оксида железа (III) из простых веществ при стандартных условиях и стандартную энтальпию образования оксида железа (III), используя следующие термохимические уравнения:

2 Fe(т)  +  O2 (г)    =   2FeO(т) ,  ∆H01  = - 527,4  кДж                     (а)

4FeO(т)  +  O2(г)  =   2Fe2O3(т) ,  ∆H02   = -587,9  кДж                     (б)

Р е ш е н и е. 1) Записываем термохимическое уравнение образования оксида железа (III) из простых веществ:

4 Fe (т) +  3O2(г) =  2Fe2O3(т) ,       ∆H03  = ? ,                                           (в)

где  H03  – тепловой эффект этой реакции.

2)  Для расчета H03 необходимо провести такую комбинацию уравнений (а) и (б), которая позволит получить уравнение реакции (в). На основании закона Гесса с термохимическими уравнениями можно оперировать также, как с алгебраическими. Для получения искомого результата следует уравнение (а) умножить на 2, а затем суммировать с уравнением (б):  4Fe + 2O2  +  4FeO +  O2 =  4FeO  +  2Fe2O3

                                        4Fe  +  3O2   =  2 Fe2O3

3) Тепловые эффекты реакции являются составной частью термохимических уравнений, поэтому с ними проведем аналогичные преобразования:
2∙∆H01  +  ∆H02  =   ∆H03

4)  Рассчитываем  H03  – тепловой эффект реакции (в):

H03  = 2(–527,4)  +  (–587,9)  =  –1054,8 – 587,9  = – 1642,7 кДж.

5) Определяем стандартную  энтальпию образования   Fe2O3  (∆H0f, 298).

Согласно уравнению (в) в результате реакции образуются 2 моль Fe2O3, поэтому

H0f, 298 (Fe2O3) = ∆H03 /2  = – 1642,7 / 2 = – 821,35 кДж/моль.

Правильность расчета проверяем, сравнивая полученное значение со справочными данными табл. 2.

Пример 2. Определите возможность самопроизвольного протекания реакции восстановления оксида хрома (III) углеродом при 298К и 1500К.

Р е ш е н и е. 1) Записываем уравнение этой реакции с указанием агрегатного состояния реагирующих веществ: Cr2O3 (т) +3C(т) = 2Cr(т) +  3CO(г)

2) Согласно условию задачи необходимо ответить на вопрос: будет ли данная реакция протекать в прямом направлении? Критерием направленности химической реакции является изменение энергии Гиббса, а условием самопроизвольного протекания реакции в прямом направлении является соотношение G < 0. Поэтому для решения задачи необходимо определить величину G.

3) Определяем, будет ли данная реакция осуществляться при Т=298К, отвечающей стандартным условиям. Рассчитываем G по уравнению (6), которое  для данной реакции имеет вид:

G0298  = (2∆G0f, 298Cr +  3∆G0f, 298  CO) – (∆G0f, 298  Cr2O3 + 3∆G0f, 298C)

Для расчета используем  значения G0f, 298  приведенные в табл.2

Так как G0f, 298  простых веществ Cr и С равны нулю, то уравнение упрощается:

G0298 = 3 моль(-137,3 кДж/моль)  1 моль(-1046,8 кДж/моль)=
= -411,9
 кДж +  1046,8 кДж  = 634,9 кДж.

Вывод: G0298 > 0, поэтому в стандартных условиях невозможно самопроизвольное протекание процесса в прямом направлении, т.е. при 298К невозможно восстановить Cr2O3  до Cr.

4) Выясняем, возможна ли данная реакция при 1500К. В условиях, отличающихся от стандартных, величина G0Т рассчитывается по уравнению (7):
GТ  = ∆H0298  TS0298. Рассчитаем тепловой эффект химической реакции при стандартных условиях, используя уравнение (3) и значения H0f, 298   из табл.2.

H0298 = (2H0f, 298 Cr + 3H0f, 298 CO) – (∆H0f, 298 Cr2O3 + 3∙∆H0f, 298  С).

Но     H0f, 298  Cr = 0  и  H0f, 298  С = 0, поэтому имеем

∆H0298 = 3∙∆H0f, 298 CO  –  ∆H0f, 298  Cr2O3,

∆H0298 = 3 моль(–110,5 кДж/моль) – 1 моль∙(–1141,0 кДж/моль) =
= −331,5
кДж + 1141,0 кДж = 809,5 кДж ,    ∆H0298  > 0,   значит реакция эндотермическая.

Определим изменение энтропии реакции при стандартных условиях. Для расчета используем уравнение (4) и значения S0 298   из табл.2.

∆S0298 = (2S0 298 Cr + 3S0 298 CO) – (S0 298 Cr2O3   + 3S0 298 С),

∆S0298 = (2 моль23,8 Дж/мольК + 3 моль197,4 Дж/мольК) – (1 моль
81,1 Дж/мольК + 3 моль5,7 Дж/мольК) = 639,8 – 98,2  = 541,6    Дж/К.

S0298  > 0, т. е. реакция сопровождается увеличением энтропии.

Рассчитаем энергию Гиббса химической реакции при Т = 1500К, т.е. величину G1500:   G1500   = ∆H0298 1500S0298 ,

G1500   = 809,5 кДж  – 1500К541,6 Дж/К = 809,5 кДж – 1500541,6 Дж.

Как видно, члены этого уравнения имеют разную размерность, поэтому приводим их к одной размерности 1 Дж = 1∙10-3 кДж и тогда имеем

G1500   = 809,5 – 1500541,6/1000 = 809,5 – 812,4  = –2,9 кДж.

Вывод: G1500 < 0, значит при 1500К данная реакция протекает самопроизвольно, и при этих условиях можно получить металлический хром.

Таблица 2

Термодинамические величины некоторых веществ в стандартных условиях:

H0f, 298 кДж/моль,  S0 298 Дж/моль∙К,  ∆G0f, 298   кДж/моль.

Вещество

H0f 298

S0 298

G0f, 298         

Вещество

H0f, 298

S0 298

G0f, 298         

Al (т)

0

+23,3

0

HCl(г)

-92,3

+187,6

-95,3

Al2O3(т)

-1677,0

+50,9

-1576,4

H2S(г)

-20,2

+205,6

-33,0

Al2(SO4)3(т)

-3434,0

+239,2

-3091,9

H2Se(г)

+86,0

+221,0

+71,0

C(т)

0

+5,7

0

H2Te(г)

+154,0

+234,0

+138,0

CO(г)

-110,5

+197,4

-137,3

Mg(т)

0

+32,0

0

CO2(г)

-393,0

+214,0

-394,0

MgO(т)

-601,2

+26,9

-569,6

Cl2(г)

0

+223,0

0

MgCO3(т)

-1096

+65,7

-1029

CaO(т)

-635,1

+29,7

-604,2

MgCl2(т)

-645,0

+90,0

-595,0

Ca(OH)2(т)

-966,2

+83,4

-896,8

N2(г)

0

+191,5

0

CaCO3(т)

-1206,0

+92,3

-1128,8

NH3(г)

-46,2

+192,5

-16,6

Cr(т)

0

+23,8

0

NO(г)

+90,4

+210,6

+86,7

Cr2O3(т)

-1141,0

+81,1

-1046,8

NO2(г)

+33,9

+240,5

+51,8

Cu(т)

0

+33,0

0

NH4Сl(т)

-315,4

+94,5

-343,6

CuO(т)

-156,0

+43,0

-127,0

O2(г)

0

+205,0

0

Fe(т)

0

+27,2

0

SO2(г)

-296,9

+248,1

-300,4

Fe2O3(т)

-821,3

+90,0

-741,0

SO3(г)

-395,2

+256,2

-370,4

H2(г)

0

+130,6

0

S(т)

0

+31,9

0

H2O(г)

-241,8

+188,7

-228,8

Ti(т)

0

+31,0

0

H2O(ж)

-285,8

+70,0

-237,5

TiCl4(г)

-759,0

+353,1

-714,0

1.6. Химическая кинетика и химическое равновесие

Химическая кинетикараздел химии, изучающий скорость и механизм химических реакций.  Различают гомогенные и гетерогенные химические реакции.

Гомогенные реакции протекают в объеме одной фазы – в жидком (водном) или газовом растворе. Например,      2СО(г) +  О2(г) = 2СО2(г)    (8)

Гетерогенные реакции протекают на границе раздела фаз: г – ж,  г – т,

т – т  и т.д.  Например,                    2С(т) + О2(г) = 2СО(г)    (9)

Скорость химической реакции (υ) – это изменение количества вещества одного из реагентов за единицу времени в единице реакционного пространства. Она зависит от природы реагирующих веществ, температуры (Т), давления (Р), концентрации (С) реагирующих веществ и других факторов. Зависимость υ от С выражается законом действующих масс: При постоянной температуре скорость химической реакции прямо пропорциональна произведению концентраций реагирующих веществ, взятых в степенях, равных стехиометрическим коэффициентам в уравнении реакции.

Например, для гомогенной реакции (8)    υ = k∙С2СО СО2 , где k – константа скорости химической реакции, которая зависит от природы реагирующих веществ, температуры, присутствия катализатора, но не зависит от концентрации реагирующих веществ; С – молярные концентрации веществ (моль / л). Для гетерогенной реакции (9)    υ = kСО2 , так как концентрации твердых веществ постоянны и приняты равными единице, т.е. СС = 1.

Зависимость скорости химической реакции от температуры выражает правило Вант-Гоффа: при повышении температуры на каждые 10º скорость химической реакции увеличивается примерно в 2 – 4 раза. Математическое выражение закона Вант-Гоффа:

,         (10)

где     υТ1   и υТ2  – скорости реакции при температурах Т1   и   Т2;  γ -  температурный коэффициент скорости химической реакции. Он показывает, во сколько раз увеличивается скорость реакции при повышении температуры на 10º.

По полноте протекания химических процессов различают необратимые и обратимые химические реакции. Необратимые реакции протекают только в одном направлении и завершаются полным превращением исходных веществ в продукты реакции. Обратимые реакции при одной и той же температуре протекают как в прямом, так и в обратном направлении и завершаются установлением в системе химического  равновесия. Они не идут до конца, в системе всегда остаются исходные вещества:

N2(г) +  3H2 (г)  2NH3(г)        (11)

Химическое равновесие – такое состояние химического взаимодействия, при котором скорости прямой и обратной реакций равны между собой,   т.е.  .

После наступления состояния равновесия концентрации реагирующих веществ при данных условиях остаются неизменными и называются равновесными концентрациями, обозначаются в квадратных скобках [ ]. Количественной характеристикой химического равновесия является константа химического равновесия К – величина, определяемая соотношением равновесных концентраций продуктов реакции и исходных веществ. Если в соотношение для константы равновесия входят равновесные концентрации, то ее обозначают символом Кс, а если используются парциальные давления газов (Рi)  , то – символом Кр.  Например, для реакции (11):

,             

Состояние химического равновесия устанавливается и сохраняется лишь при определенных условиях (температуре, давлении и концентрации реагирующих веществ).  При изменении хотя бы одного из этих условий, т.е. при каком-либо внешнем воздействии, равновесие в системе нарушается,  и система переходит в новое состояние равновесия. Этот переход называется смещением химического равновесия.

Влияние внешних воздействий (изменение Т, Р или С) на состояние химического равновесия можно предсказать, пользуясь принципом Ле Шателье-Брауна: если на систему, находящуюся в равновесии, оказать внешнее воздействие, то равновесие смещается в том направлении, которое ослабляет эффект внешнего воздействия, т.е на внешнее воздействие система отвечает противодействием. Так увеличение концентрации какого-либо вещества вызовет противодействие системы – стремление уменьшить концентрацию этого вещества, что возможно путем смещения равновесия в направлении, при котором концентрация этого вещества будет уменьшаться.

Повышение Робщ или Рi; приводит к смещению равновесия в сторону протекания той реакции, которая вызывает уменьшение Робщ и Рi  и наоборот, понижение Робщ или Pj смещает равновесие в направлении той реакции, которая приводит к увеличению этих параметров.

Повышение температуры вызывает смещение равновесия в направлении протекания той реакции, которая сопровождается поглощением теплоты, т.е. эндотермической (∆Н > 0). Понижение температуры будет вызывать смещение равновесия в сторону экзотермической реакции, сопровождающееся выделением теплоты (∆Н < 0).

Примеры решения задач

Пример 1. Как изменится скорость реакции  2NО(г) + О2(г) ⇄ 2NO2(г),  если   
одновременно концентрацию
увеличить в 2 раза, а концентрацию О2 уменьшить в 8 раз?

Р е ш е н и е. 1) Зависимость скорости химической реакции от концентрации реагирующих веществ выражается законом действующих масс, поэтому записываем математическое выражение этого закона для данной реакции в начальный момент времени:

Согласно условию концентрацию NO увеличили в 2 раза, т.е. С'NO=2СNO, а концентрацию О2 уменьшили в 8 раз, т.е.   С'o2 = Сo2 /8.

2) Записываем выражение для скорости реакции  в конечный момент времени:  υ' = k (С' NO)2  С'o2  или  

3) Определяем, как изменяется скорость реакции. Для этого рассматриваем  отношение          =  1/2

  Ответ: при одновременном увеличении концентрации в 2 раза и уменьшении концентрации О2 в 8 раз скорость реакции уменьшается в 2 раза.

Пример 2. Определите, во сколько раз изменятся скорости прямой и обратной реакций в системе   2SO2 (г) + О2 (г) 2SО3 (г), если объем газовой смеси уменьшить в 3 раза. Каково направление смещения равновесия в этой системе?

Р е ш е н и е. 1) Записываем выражения для скоростей прямой и обратной реакций, используя закон действующих масс:   ,   

2) Рассматриваем, что происходит в реакционной системе при уменьшении  ее объема. Уменьшение объема гомогенной системы в 3 раза приводит к увеличению концентрации каждого из реагирующих веществ также в 3 раза, т.е. математически это записываем в виде: С'so2 = 3С so2 ,  С'о2 = 3Сo2   и  С'so3 = 3С so3.

3) Записываем выражение для скоростей прямой и обратной реакций после изменения объема системы:

4) Определяем, во сколько раз изменяются скорости прямой и обратной реакций:   ;                          

Таким образом, при уменьшении объема газовой смеси в 3 раза скорость прямой реакции увеличится в 27 раз, а скорость обратной реакции – в 9 раз.

5) Устанавливаем направление смещения равновесия в этой системе. В связи с тем, что скорость прямой реакции увеличилась в 27 раз, а обратной – только в 9 раз, равновесие сместится в направлении протекания прямой реакции, т.е. в сторону образования SO3.

Пример 3. Рассчитайте температурный коэффициент скорости химической реакции, если известно, что константа скорости этой реакции при 140°С равна 5,5 10-4, а при 185°С  – 9,2 10-3.

Дано : Т1 =140° С, k1 =5,5 10 -4 ; Т2= 185°С, k2  = 9,2∙ 10-3. Найти γ.

Р е ш е н и е. 1) Зависимость скорости химической реакции от температуры описывается правилом Вант-Гоффа в форме:

Но для проведения расчетов это уравнение необходимо прологарифмировать, т.е.   

2) Рассчитываем температурный коэффициент скорости этой реакции.

а) вычисляем температуру по абсолютной шкале:

T1 = 140 + 273 = 413 К,                    Т2 = 185 + 273 = 458 К;

б) рассчитаем численное значение γ:

, lg 16,7 = 4,5 lgγ

отсюда , т.е. lg γ = 0,27, и тогда γ = 1,87.

Ответ: температурный коэффициент этой реакции равен 1,87.

Пример 4. При синтезе аммиака N2 + ЗН2  2NНз равновесие установилось при следующих концентрациях реагирующих веществ (моль/л) [N2 ] = 4; [H2] = 2 и [NH3] = 6. Рассчитайте константу равновесия и исходные концентрации Н2 и N2 .

Дано: [Н2]=2 моль/л, [N2] = 4 моль/л, [NH3] = 6 моль/л. Найти Кс, Сисх(Н2), Cиcx(N2).

Р е ш е н и е. 1) Записываем выражение для константы равновесия в этой системе и рассчитываем ее значение : ;          

2) Определяем исходные концентрации Н2 и N2.

Для обратимых реакций ни одно из реагирующих веществ не расходуется полностью. Поэтому исходная концентрация вещества А складывается из равновесной концентрации этого вещества [А] и концентрации этого вещества, которая была израсходована на реакцию к моменту установления равновесия  (условно обозначим ее X (А) . Таким образом, выражение для исходной концентрации вещества А может быть представлено в виде:    Сисх(А) = [А] +  Х(А).

а) Для определения Сисх(N2) и Сисх2) обращаемся к уравнению реакции, согласно которому на образование 2 моль NH3 требуется 1 моль N2 и 3 моль Н2. Но поскольку равновесная концентрация  NH3 равна 6 моль/л, то при этом было израсходовано  6/2 = 3 моль N2   и 3(6 / 2) = 9 моль Н2.

б)  Рассчитаем   Сисх(N2) и Сисх2):

Сисх2) = [Н2] + Х(Н2) = 2 + 9 =  11 моль/л,

Cисх(N2) = [N2] + X(N2) = 4 + 3 = 7 моль/л.

Ответ: константа равновесия реакции равна 1,1. Исходные концентрации Н2 и N2 составляют 11 моль/л  и 7 моль/л  соответственно.

Пример 5. Установите направление смещения равновесия в системе

2О(г)+ 2О2(г)2О2 (г),        ∆ Н = – 483,7 кДж

при понижении температуры и уменьшении общего давления.

Р е ш е н и е. Влияние внешнего воздействия на состояние химического равновесия в системах, в которых протекают обратимые реакции, оценивается с помощью принципа Ле Шателье-Брауна.

1) Рассматриваем влияние температуры на состояние равновесия в данной системе.  Термохимическое уравнение реакции показывает, что ∆Н < 0, следовательно, прямая реакция экзотермическая. При понижении температуры согласно принципу Ле Шателье-Брауна система будет стремиться ослабить внешнее воздействие (т.е. повысить Т), а это возможно за счет реакции, протекающей  с выделением теплоты, т.е. экзотермической реакции. Следовательно, понижение температуры приведет к смещению равновесия в направлении протекания прямой реакции.

2) Оцениваем влияние изменения давления на состояние равновесия в данной системе. Из уравнения реакции видно, что прямая реакция сопровождается уменьшением объема системы 2V (Н2О) + 2V (О2)  ≠  2V (Н2О2),  т.е. ∆V ≠ 0. Поэтому общее давление (Робщ) будет влиять на состояние равновесия в данной системе. Уменьшение Робщ согласно принципу Ле Шателье-Брауна вызовет противодействие системы (увеличение Робщ) за счет смещения равновесия в сторону протекания реакции, сопровождающейся увеличением Робщ, т.е. в сторону обратной реакции. Действительно, обратная реакция приводит к увеличению числа моль газообразных веществ в системе, а это значит, что при протекании обратной реакции Робщ  в системе увеличивается.

1.7. Электролитическая диссоциация. Реакции  ионного обмена

Электролитическая диссоциация – это процесс распада молекул электролита на ионы под действием полярных молекул растворителя.

Электролиты – это вещества, расплавы или водные растворы которых проводят электрический ток. К ним относятся растворы кислот, расплавы и растворы щелочей и солей. Неэлектролиты – это вещества, которые не проводят электрический ток. К ним относятся многие органические вещества.

Электролиты, которые практически полностью диссоциируют на ионы, называются сильными; электролиты, которые частично диссоциируют на ионы, называются слабыми. Для количественной оценки полноты диссоциации введено понятие степени диссоциации. Степенью диссоциации электролита называют отношение числа молекул, распавшихся на ионы, к общему числу молекул, находящихся в растворе.

Обычно степень диссоциации (α) выражают в долях единицы или %:

,     (12)

где n – число частиц, подвергшихся электролитической диссоциации;

n0 – общее число частиц в растворе.

 Сильные электролиты – почти все соли, растворимые основания ( NaOH, KOH, Ba(OH)2 и др.), неорганические кислоты (H2SO4, HCl, HNO3, HBr, HI и др).

Слабые электролиты – нерастворимые основания и NH4OH, неорганические кислоты (H2CO3,, H2S, HNO2,  H3PO4   и др.), органические кислоты и  вода H2O.

Сильные электролиты диссоциируют на ионы практически нацело (т.е. процесс диссоциации является необратимым)   и одностадийно:

HCl = H+ + Cl–                                        H2SO4 = 2H+ + SO42–

Слабые электролиты диссоциируют частично (т.е. процесс диссоциации является обратимым)  и ступенчато. Например, для многоосновных кислот на каждой стадии происходит отрыв одного иона водорода:

1.  H2SO3  H+ + HSO3-           2.  HSO3- H+ + SO32-

Таким  образом, число стадий многоосновных кислот определяется основностью кислоты (числом ионов водорода), а число стадий многокислотных оснований будет определяться кислотностью основания (или числом гидроксильных групп): NH4OH  NH4+ + OH. Процесс электролитической диссоциации завершается установлением в системе состояния химического равновесия, которое характеризуется константой равновесия:

Константа равновесия процесса электролитической диссоциации называется константой диссоциации – КД. Константа диссоциации зависит от природы электролита,  природы растворителя, температуры, но не зависит от концентрации электролита.

Между КД и α существует количественная связь:

     (13)

Соотношение (13) называют законом разбавления Оствальда: степень диссоциации слабого электролита возрастает с разбавлением раствора.

Для слабых электролитов, когда α 1,  КД = α2С.

Вода является слабым электролитом, поэтому диссоциирует обратимо:

H2O  H+ + OH    ∆H = +56,5кДж/моль

Константа диссоциации воды:  

Степень диссоциации воды очень мала (это очень слабый электролит). Так как вода присутствует в большом избытке, то ее концентрация может считаться величиной постоянной и составляет , тогда

КД [H2O] = [H+]∙[OH-] = 55,6∙1,8∙10-16 = 10-14

[H+]∙[OH-] = 10-14 = KW – ионное произведение воды

Так как в  воде концентрации  катионов водорода и гидроксид-ионов  равны, то: [H+] = [OH-] = .

Растворение в воде других веществ (кислот, оснований, солей) изменяет концентрацию ионов Н+ или ОН, а их произведение всегда остается постоянным и равным 10-14 при Т=250С. Концентрация ионов Н+ может служить мерой кислотности или щелочности раствора. Обычно для этой цели используется водородный показатель: pH = -lg[H+]. Таким образом, водородный показатель – это десятичный логарифм концентрации ионов водорода, взятый с обратным знаком.

В зависимости от концентрации ионов водорода различают три среды.

В нейтральной среде [H+] = [OH-]= 10-7 моль/л, рН= – lg 10-7 = 7. Эта среда характерна как для чистой воды, так и для нейтральных растворов. В кислых растворах [H+] > 10-7моль/л,  рН < 7.  В кислых средах рН меняется в пределах  0 < рН < 7. В щелочных средах [H+] < [ОН] и [H+] < 10-7 моль/л, следовательно,  рН > 7. Пределы изменения рН: 7 < рН < 14.

Реакции ионного обмена (РИО) – это реакции между ионами, протекающие в водных растворах электролитов. Отличительная особенность обменных реакций: элементы, входящие в состав реагирующих веществ, не меняют свою степень окисления. Реакции ионного обмена являются необратимыми реакциями и протекают при условии: 1) образования малорастворимого вещества, 2) выделения газообразного вещества, 3) образования слабого электролита.

При протекании РИО противоположно заряженные ионы связываются и выводятся из сферы реакции. Сущность реакций ионного обмена выражают с помощью ионных уравнений, которые, в отличие от молекулярных, показывают истинных участников реакции. При составлении ионных уравнений следует руководствоваться тем, что вещества малодиссоциирующие, малорастворимые (выпадающие в осадок) и газообразные записываются в молекулярной форме. Сильные растворимые электролиты записываются в виде ионов.  Поэтому при написании ионных уравнений необходимо пользоваться таблицей растворимости солей и оснований в воде.

Гидролиз – это процесс взаимодействия ионов соли с молекулами воды, приводящий к образованию малодиссоциирующих соединений; является частным случаем реакций ионного обмена.  Гидролизу подвергаются соли, образованные:

  1.  слабой кислотой и сильным основанием (NaCH3COO, Na2CO3 , Na2S,);
  2.  слабым основанием и сильной кислотой (NH4Cl, FeCl3,, AlCl3,);
  3.  слабым основанием и слабой кислотой (NH4CN, NH4CH3COO).

Соли, образованные сильной кислотой и сильным основанием, гидролизу не подвергаются: Na2SO4, BaCl2, NaCl, NaJ и т.д.

Гидролиз солей увеличивает концентрации ионов Н+ или ОН. Это приводит к смещению ионного равновесия воды и в зависимости от природы соли сообщает раствору кислую или щелочную среду (см. примеры решения задач).

Примеры решения задач

Пример 1. Вычислить константу диссоциации уксусной кислоты СН3СООН, зная, что в 0,1М растворе она диссоциирована на 1,32%.

Р е ш е н и е. Для решения задачи воспользуемся законом разбавления Оствальда – уравнение (13),  в котором С – концентрация уксусной кислоты, – степень ее диссоциации в данном растворе, значение которой равно 1,32/100=0,0132. Таким образом,     

Ответ: константа диссоциации уксусной кислоты составляет 1,7610-5.

Пример 2. Вычислить значение рН 0,1М раствора гидроксида аммония NH4OH, приняв степень диссоциации раствора равной 1%.

Р е ш е н и е. 1)  Записываем уравнение диссоциации раствора NH4OH:

NH4OH  NH4+ + OH

2) Так как гидроксид аммония – слабый электролит и процесс его диссоциации носит обратимый характер, к моменту равновесия в 1 л раствора продиссоциировало С∙ моль NH4OH (С = 0,1 моль/л) и образовалось столько же моль ОН – ионов:  
[OH-] = С∙ = 0,1∙0,01 = 110-3 моль/л, следовательно, рОН = 3

3)   Рассчитываем значение рН:   pH + pOH = 14, рН = 14 – 3 = 11.

Ответ: рН 0,1М раствора гидроксида аммония равно 11.

Пример 3. Выразить с помощью ионного уравнения сущность реакции:

Pb(NO3)2 + KI 

Р е ш е н и е. 1)  Составляем молекулярное уравнение реакции:

Pb(NO3)2 + 2KI  PbI2 + 2KNO3

Отмечаем, что в результате образуется нерастворимое вещество PbI2.

2) Составляем полное ионное уравнение (все сильные электролиты записываем в виде ионов, слабые, уходящие из сферы реакции, – в виде молекул):

Pb2+ + 2NO3 + 2K+ + 2I = PbI2 + 2K+ + 2NO3

3) Составляем сокращенное ионное уравнение, в котором исключаем все ионы, повторяющиеся в обоих частях уравнения, т.е. ионы не участвующие в реакции:

Pb2+ + 2NO3 + 2K+ + 2I = PbI2 + 2K+ + 2NO3

Pb2++ 2I = PbI2

Из сокращенного ионного уравнения видно, что сущность реакции сводится к взаимодействию ионов Pb2+ и I, в результате которого образуется труднорастворимое вещество PbI2.

Пример 4. Определить характер среды водного раствора соли ZnCl2.

Р е ш е н и е. ZnCl2 – соль образованная сильной кислотой HCl и слабым основанием Zn(OH)2, следовательно, подвергается гидролизу по катиону. Так как  катион двухзарядный, гидролиз будет протекать по двум ступеням.

I ступень:

а) записываем уравнение процесса диссоциации ZnCl2:

ZnCl2 = Zn2+ + 2Cl

Подчеркнем ион, по которому идет гидролиз, это ион Zn2+.

б) процесс взаимодействия иона Zn2+ с водой происходит по схеме:

Zn2+ +  H+–OH  (ZnOH)+ + H+

в) запишем суммарное уравнение гидролиза:

ZnCl2 = Zn2+ + 2Cl

Zn2+ + H2O (ZnOH)+ + H+

ZnCl2 + Zn2+ + H2O Zn2+ + (ZnOH)+ + 2Cl + H+

После преобразований получаем:

ZnCl2 + H2O Zn(OH)Cl + HCl

В результате гидролиза образуется сильная кислота HCl, поэтому  рН < 7.

II ступень:

При рассмотрении гидролиза по второй ступени используется аналогичный подход.

а) Zn(OH)Cl = (ZnOH)+ + Cl

б) (ZnOH)+ + H+OH = Zn(OH)2 + H+

в) Zn(OH)Cl = (ZnOH)+ + Cl

(ZnOH)+ + H2O = Zn(OH)2 + H+

Zn(OH)Cl + H2O = Zn(OH)2 + HCl

Таким образом, в результате гидролиза по второй ступени образуется сильная кислота HCl, которая создает кислую среду раствора (рН < 7).

1.8. Растворы. Способы выражения концентрации растворов

Раствор – гомогенная (однородная) система, состоящая как минимум из двух компонентов,  один из которых растворитель, другой – растворенное вещество. То есть состав раствора = растворитель + растворенное вещество. Например, водный раствор хлорида натрия состоит из двух компонентов: воды (растворителя) и хлорида натрия (растворенного вещества).

Существует несколько способов выражения концентрации растворов.

Молярная концентрация, или молярность (СM) – количество вещества (ν) растворенного компонента, содержащееся в 1 литре (дм3) раствора:

[моль/л],       (14)

Молярная концентрация эквивалентов (Сэкв), (нормальная концентрация или нормальность N) – количество вещества эквивалентов (νэкв) растворенного компонента, содержащееся в 1 литре (дм3) раствора:

                 Сэкв (N)   =  νэкв   / V    [моль-экв/л]      (15)

Моляльная концентрация или  моляльность (b) – количество вещества (ν) растворенного компонента в 1 кг раствора:

[моль/кг],       (16)

Массовая доля (ω) – отношение массы растворенного вещества (mр.в.) к массе раствора (mр-ра) Ее рассчитывают, выражая в долях единицы или в процентах. Массовая доля, выраженная в процентах, называется процентной концентрацией: 

      (17)

Процентная концентрация показывает массу растворенного вещества, содержащегося в 100 г раствора. Например, ω(KOH) = 3% означает, что в 100 г этого раствора содержится 3 г KOH и 97 г H2O.

Молярная доля (Ni) – отношение количества вещества растворенного компонента (в-ва) (или растворителя, р-ля) к суммарному  количеству вещества всех компонентов раствора. Например, в системе, состоящей из растворителя и одного растворенного вещества, молярная доля растворенного вещества равна:

,       (18)

Молярная доля растворителя:

      (19)

Примеры решения задач

Пример 1. Определить молярную концентрацию раствора NaOH с массовой долей 10%  и  плотностью =1,1 г/см3.

Р е ш е н и е. 1) Записываем выражение для молярной концентрации раствора NaOH : 

2) 10%-ный раствор – это 10г NaOH в 100г раствора. Находим количество вещества NaOH, содержащееся в 10 г:

     ,   mNaOH = 10 г,    MNaOH = 23 + 16 +1 = 40 г/моль

, т.е. 0,25 моль NaOH содержится в 100 г раствора.

3) Находим объем раствора массой 100 г:

m=V,    = 0,091 л

4) Рассчитываем молярную концентрацию:

Ответ: молярная концентрация раствора гидроксида натрия с массовой долей 10% составляет 2,74 моль/л

Пример 2. Найти молярную долю растворенного вещества в растворе сахарозы с массовой долей 67%.

Р е ш е н и е. 1) Вспомним, что молярная доля растворенного вещества равна:

67%-ный (по массе) раствор означает, что в 100 г раствора содержится  67 г сахарозы и 33 г воды.

2) Определяем количество вещества сахарозы и количество вещества воды:

в-ва = 67/342 = 0,196 моль,                  р-ля = 33/18 = 1,83 моль

Следовательно, молярная доля сахарозы равна:

Ответ: молярная доля сахарозы в растворе сахарозы с массовой долей 67% составляет 0,097.

Пример 3. Какой объем серной кислоты с массовой долей 96% (плотностью 1,84 г/см3) и какую массу воды нужно взять для приготовления 100 мл 15%-ного  (по массе) раствора H2SO4  ( = 1,10 г/см3).

Р е ш е н и е. 1)  Найдем массу 100 мл 15% раствора H2SO4:

mH2SO4-р-ра = V = 1001,10 =110 г

2)  Из формулы массовой доли находим массу серной кислоты, содержащейся в этом растворе:

;         mH2SO4 =

3)  Найдем массу 96% раствора, содержащего 16,5 г H2SO4:

4)  Находим объем 96 %-ного раствора серной кислоты:

Ответ: для приготовления 100мл 15%-ного раствора H2SO4 потребуется 9,3 мл 96%-ного раствора серной кислоты и 110 – 16,5 = 93,5 г воды.

1.9. Коллоидные растворы

Дисперсные системы - это системы, состоящие как минимум  из двух веществ, одно из которых измельчено и распределено в другом.

То вещество, которое образует в дисперсной системе сплошную фазу, называют дисперсионной средой, а то, что распределено в среде – дисперсной фазой. Гомогенные дисперсные системы называют истинными растворами или просто растворами. Линейные размеры их частиц не превышают размеров отдельных ионов и молекул – до 1 нм. Гетерогенные дисперсные системы подразделяются на коллоидные системы (размеры частиц от 1 до 100 нм) и грубодисперсные или микрогетерогенные системы (размеры частиц более 100 нм).

Коллоидные системы, дисперсионная среда которых жидкость, называются коллоидными растворами или золями. Их можно рассматривать как частный случай истинных растворов. Дисперсная фаза – это растворенное вещество, а дисперсионная среда – растворитель.

Для получения коллоидных растворов используются любые реакции, в результате которых образуются труднорастворимые  соединения:

FeCl3 + 3H2O = Fe(OH)3↓ + 3HCl       (20)

AgNO3 + KI = AgI↓ +KNO3        (21)

Ba(Cl)2 + Na2SO4 = BaSO4↓+ 2NaCl      (22)

Структурной единицей коллоидного раствора является мицелла – это отдельная частица дисперсной фазы с жидкой дисперсионной средой. Рассмотрим образование мицеллы на примере реакции (21). Избыток одного из компонентов действует как стабилизатор коллоидного раствора, то есть как вещество, препятствующее агрегации коллоидных частиц в более крупные и выпадению их в осадок.

Пусть в избытке будет азотнокислое серебро. Труднорастворимый AgI образует кристаллический агрегат, состоящий из  m молекул AgI. Агрегат адсорбирует на поверхности ионы Ag+, находящиеся в избытке. Они придают агрегату положительный заряд и называются потенциалопределяющими ионами. Агрегат и потенциалопределяющие ионы образуют ядро (m AgIn Ag+. С заряженной поверхностью ядра устойчиво связано некоторое число ионов противоположного знака – противоионов(n-x)NO3. Потенциалопределяющие ионы и связанные противоионы образуют адсорбционный слой. Агрегат вместе с адсорбционным слоем называется гранулой или коллоидной частицей. Она имеет электрический заряд, совпадающий с зарядом потенциалопределяющего иона (х+). В состав коллоидной частицы входит только часть имеющихся в растворе противоионов. Остальные противоионы  xNO3-  остаются в дисперсионной среде и образуют диффузионный слой. Заряды потенциалопределяющих ионов и противоионов полностью скомпенсированы. Поэтому мицелла электронейтральна.

Строение мицеллы золя иодида серебра имеет вид:

        ядро

{[mAgI]nAg+(n-x)NO3-}x+ ∙ xNO3-

агрегат    адсорбционный   диффузионный

                     слой                      слой

Если в растворе избыток  KI, то мицелла будет иметь вид:

{[mAgI]nI-(n-x)K+ }x-xK+

Строение мицеллы золя сульфата бария, полученного по реакции (22) с избытком хлорида бария: {[m BaSO4] n Ba2+ 2(n-x)Cl-}2x+ 2xCl-

Строение мицеллы золя Fe(OH)3:  {[m (FeOH)3] nFe3+  3(n-x)Cl- }3x+ 3xCl-

Устойчивость – способность коллоидных систем сохранять свое состояние и свойства неизменными с течением времени. Различают два вида устойчивости: кинетическую (седиментационную) и агрегативную. Кинетическая устойчивость характеризует способность частиц дисперсной фазы оставаться во взвешенном состоянии благодаря интенсивному броуновскому движению. Агрегативная устойчивость характеризует способность частиц дисперсной фазы противостоять их агрегации, т.е. укрупнению и слипанию. Это обусловлено наличием одноименного электрического заряда частиц дисперсной фазы, вызывающего их взаимное отталкивание. Устойчивость золя можно нарушить, устранив одноименный заряд коллоидных частиц. Это можно сделать при добавлении электролита. Потеря агрегативной устойчивости золя приводит к укрупнению частиц дисперсной фазы , их слипанию Этот процесс называют коагуляцией. Коагуляция вызывает нарушение кинетической устойчивости, которое приводит к образованию осадка (коагулята). Этот процесс называется седиментацией.

Коагулирующим действием обладает тот ион, который заряжен противоположно частице. Коагулирующая способность электролита возрастает с увеличением заряда коагулирующего иона. Например, для золя иодида серебра коагулирущим  действием будут обладать такие анионы, как Cl-, SO42-, PO43-. Из них наилучшим коагулирующим действием обладает ион PO43-. Процесс коагуляции начинается только после достижения определенной минимальной концентрации электролита, которая называется порогом коагуляции.

Примерный состав коагулята золей иодида серебра и гидроксида железа:

    {[mAgI]nAgnNO3-}0    ,                { [m (FeOH)3] nFe3+  3nCl- }0

Примеры решения задач

Пример 1. Золь иодида серебра AgI получен при добавлении к 0,02 л 0,01 М раствора KI   0,028 л  0,005М раствора AgNO3. Определите заряд частиц полученного золя и напишите формулу его мицеллы.

Д а н о: СKI = 0,01 моль/л; СAgNO3 = 0,005 моль/л, VKI = 0,02 л; V AgNO3 = 0,028 л.

Определить заряд полученного золя. Написать формулу мицеллы золя.

Р е ш е н и е. При смешении растворов AgNO3 и KI  протекает реакция: AgNO3 + KI = AgI + KNO3

Определяем количество AgNO3 и KI, участвующих в реакции:

СVAgNO3 = 0,0050,028 = 1,410-4 моль

CVKI = 0,020,01 = 2,010-4  моль

Расчет показывает, что в растворе избыток KI, следовательно, ядром коллоидных частиц золя иодида серебра будут адсорбироваться ионы I и частицы золя приобретают отрицательный заряд. Противоионами являются ионы К+. Формула мицеллы золя иодида серебра при условии избытка KI:

[mAgI] nI– (n–x)K+}–x xK+

Пример 2. Какой объем 0,002 М раствора BaCl2 надо добавить к 0,03 л 0,0006 М раствора Al2(SO4)3, чтобы получить положительно заряженные частицы золя сульфата  бария. Напишите формулу мицеллы золя  BaSO4.

Д а н о: СBaCl2 = 0,002 моль/л; С Al2(SO4)3 = 0,0006 моль/л, V Al2(SO4)3 = 0,03 л.

Найти VBaCl2. Написать формулу мицеллы золя.

Р е ш е н и е.  Образование золя BaSO4 происходит в соответствии с уравнением реакции:  3BaCl2 + Al2(SO4)3 = 3BaSO4 + 2AlCl3

Если вещества участвуют в стехиометрическом соотношении, то справедливо соотношение: (CV)BaCl2 = (CV)Al2(SO4)3,  следовательно

VBaCl2 =

Для получения положительных частиц золя BaSO4 в растворе должен быть избыток хлорида бария  по сравнению с сульфатом алюминия. Значит, для реакции необходимо взять более 0,009 л   0,002М раствора BaCl2. Формула мицеллы золя сульфата бария:    [mBaSO4] nBa2+ 2(n-x)Cl }2x+2xCl

Пример 3. Золь сульфида цинка был получен при взаимодействии растворов Zn(NO3)2 и Na2S. Определите, какой из электролитов был в избытке, если противоионы в электрическом поле движутся к аноду. Напишите формулу мицеллы золя.

Р е ш е н и е. Образование золя ZnS происходит в соответствии с уравнением реакции:    Zn(NO3)2 + Na2S = ZnS + 2NaNO3

Анод – положительно заряженный электрод, к нему движутся отрицательно заряженные частицы. Значит противоионы мицеллы  имеют отрицательный заряд, а сама мицелла заряжена положительно, что возможно при условии избытка Zn(NO3)2. На поверхности агрегата ZnS адсобируются потенциалопределяющие ионы Zn2+ (так как раствор Zn(NO3)2 в избытке), создавая таким образом положительный заряд мицеллы. Агрегат и потенциалопределяющие ионы образуют ядро, с которым связываются противоионы – NO3.

Таким образом, в избытке был взят раствор Zn(NO3)2. Формула мицеллы золя иодида серебра при условии избытка Zn(NO3)2:

[m ZnS ] nZn2+ 2(n-x)NO32x+2xNO3

1.10. Растворы неэлектролитов

Растворы неэлектролитов состоят из незаряженных частиц. Они могут быть образованы различными парами органических жидкостей, например бензолом и толуолом.

Установлено, что молекулы нелетучего растворенного компонента раствора препятствуют улетучиванию из раствора молекул растворителя. Согласно закону Рауля  понижение давления P  насыщенного пара растворителя над раствором  пропорционально мольной доле растворенного нелетучего вещества ni:

P = P0 P = P0 ni, или   ,    (23)

где Р0 – давление насыщенного пара растворителя над чистым растворителем; P давление насыщенного пара растворителя над раствором, P – разность между давлениями насыщенного пара растворителя над раствором P и растворителем Р0;  n – количество растворенного вещества в растворе (моль); N – количество вещества растворителя (моль); ni – мольная доля растворенного вещества.

Из закона Рауля возникают два следствия.

1. Температура кипения раствора выше температуры кипения растворителя.  Повышение температуры кипения Ткип пропорционально моляльной концентрации раствора сm:

Ткип = Кэсm,          (24)

где  Кэ – эбулиоскопическая постоянная растворителя.

         (25)

где g – масса растворенного вещества, г; G – масса растворителя, г;
Mr – молярная масса растворенного вещества.

2. Температура замерзания раствора ниже температуры замерзания чистого растворителя. Понижение температуры замерзания Тзам пропорционально моляльной концентрации раствора сm:

Тзам = Кксm,          (26)

где Кк – криоскопическая постоянная.

Значения Кэ и Кк зависят от природы растворителя. Используя уравнения
(23 - 25), можно определить молярную массу вещества
Mr:

,          (27)

где Т – изменение температуры кипения или замерзания раствора;
К  Кэ или К  Кк

Самопроизвольный переход растворителя через полупроницаемую мембрану, разделяющую раствор и растворитель или два раствора с различной концентрацией растворенного вещества, называется осмосом. Осмос обусловлен диффузией молекул растворителя через полупроницаемую перегородку (мембрану), которая пропускает только молекулы растворителя. Количественно осмос характеризуется осмотическим давлением, равным силе, приходящейся на единицу площади поверхности, и заставляющей молекулы растворителя проникать через полупроницаемую перегородку.

Осмотическое давление возрастает с увеличением концентрации растворенного вещества и температуры. Вант-Гофф предположил, что для осмотического давления можно применить уравнение состояния идеального газа:

V=nRT ;       ;         RT,       (28)

где  – осмотическое давление (Па); n – количество вещества (моль); V – объем раствора (м3); С – молярная концентрация раствора, R – универсальная газовая постоянная, 8,3144 Дж/(мольК).

Примеры решения задач.

Пример 1. Вычислите осмотическое давление раствора, содержащего в 1,4 л
63 г глюкозы С
6Н12О6 при 00С.

Д а н о: m = 63 г, V = 1,4 л, Т = 00С. Найти .

Р е ш е н и е. Осмотическое давление раствора определяют согласно закону Вант-Гоффа: . Зная массу глюкозы, можно найти ее количество вещества n: молярная масса глюкозы равна 180,16 г/моль, , следовательно, в 1,4 л раствора содержится 0,35 моль глюкозы. Осмотическое давление этого раствора :

Пример 2. Определите температуру кипения и замерзания раствора, содержащего 1 г нитробензола С6Н5NO2 в 10 г бензола. Эбулиоскопическая и криоскопическая константы соответственно равны 2,57 и 5,10С. Температура кипения чистого бензола 80,20С,  температура замерзания –5,40С.

Д а н о: g = 1 г, G = 10 г, Кэ = 2,570С, Кк = 5,10С, Тк бенз = 80,20С, Тз бенз = –5,40С.

Найти Тк р-ра и Тз р-ра 

Р е ш е н и е 1)  Из закона Рауля следует, что

,   

2) Рассчитаем молярную массу нитробензола:

Mr (C6H5NO2) = 123,11 г/моль.

3) Повышение температуры кипения раствора нитробензола в бензоле:

4) Температура кипения раствора: Тк = 80,2 + 2,09 = 82,290С.

5) Понижение температуры замерзания раствора нитробензола в бензоле:

6) Температура замерзания раствора: Тз = 5,4 – 4,14 = 1,260С.

Пример 3. Раствор камфоры массой 0,552 г в 17 г эфира кипит при температуре на 0,4610С выше, чем чистый эфир. Эбулиоскопическая постоянная эфира 2,160С. Определите молекулярную массу камфоры.

Д а н о: g = 0,552 г, G = 17 г, Кэ = 2,160С. Найти Mr камфоры

Р е ш е н и е.

Молекулярную массу камфоры можно определить, пользуясь соотношением

Таким образом, молекулярная масса камфоры равна 155,14 г/моль.

Пример 4. Вычислите давление пара над раствором, содержащим 34,23 г сахара С12Н22О11, в 45,05 г воды при 650С, если давление паров воды при этой температуре равно 2,5104 Па.

Д а н о: m12Н22О11) = 34,23 г, m (H2O) = 45,05 г, Р0 = 2,5104 Па. Найти Р.

Р е ш е н и е. 1) Относительное понижение давления пара растворителя над раствором согласно закону Рауля выражается соотношением     

2) Рассчитаем молярные массы сахара и воды:

М 12Н22О11) = 342,30 г/моль, М (Н2О) = 18,02 г/моль.

3) Найдем количество растворенного вещества и растворителя:

,

4) Давление пара над раствором:

1.11.  Окислительно-восстановительные реакции

Окислительно-восстановительными реакциями (ОВР) называют химические процессы, сопровождающиеся переносом электронов от одних молекул или ионов к другим, в результате чего происходит изменение степени окисления элементов, входящих в соединения.

Степень окисления (С.О.)– это условный заряд атома элемента, вычисленный из предположения, что все связи в молекуле соединения являются ионными. При вычислении С.О. какого-либо элемента следует знать те элементы, которые в своих соединениях проявляют всегда одну степень окисления (табл. 3). В любой окислительно-восстановительной реакции имеют место два взаимосвязанных процесса: окисление и восстановление.

Окисление – процесс отдачи электронов атомом, молекулой или ионом. При окислении С.О. элемента повышается: Al0 – 3e-  Al3+. Восстановление – процесс присоединения электронов атомом, молекулой или ионом. В этом случае С.О. элемента понижается:  Fe3+ + 1 e-  Fe2+.

Восстановители – это частицы (атомы, молекулы, ионы) которые отдают электроны, сами при этом они окисляются. Окислители – это частицы, которые принимают электроны, сами при этом они восстанавливаются.

Таблица 3.

Элементы с постоянной степенью окисления

С.О.

Элементы

0

Простые вещества

+1

H (кроме гидридов: CaH2, NaH и др.),

Щелочные металлы: Li, Na, K, Rb, Cs, Fr

+2

Щелочно-земельные металлы: Be, Mg,Ca, Ba, Sr,

а также d-элементы: Zn, Cd

+3

B, Al

-2

O (кроме H2O2-1  и O+2F2)

-1

F

Рассмотрим алгоритм составления уравнений ОВР методом электронного баланса, суть которого заключается в уравнивании числа электронов, отданных восстановителем, и числа электронов, принятых окислителем.

1. Записывают молекулярное уравнение реакции, например:

 Br2  +  H2S  + H2OHBr  +  H2SO4

2. Определяют степени окисления атомов всех элементов в исходных веществах и продуктах реакции:

 Br20 +  H2+1S-2  + H2+1O-2 → H+1Br -1  +  H2+1S+6O4-2

  

3. Находят элементы, атомы которых изменяют степени окисления. В данном случае это S и Br.

4. Определяют окислитель и восстановитель. Записывают электронные схемы окисления и восстановления. Определяют процессы окисления и восстановления. Подбирают коэффициенты, на которые следует умножить уравнения электронного баланса перед их суммированием.

S-2 – 8ē   S+6           1 – окисление

Br20 + 2ē 2Br -1      4 – восстановление

S-2 + 4Br20  S+6 + 8Br -1      

Z = –2+ 4(0) = –2          Z = +6+8 (–1) = –2

5. Составляют суммарное уравнение электронного баланса, в котором расставляют коэффициенты с учетом числа принятых и отданных электронов. Проверяют равенство зарядов: сумма зарядов в левой количества уравнения должна совпадать с суммой зарядов в правой части.

6. Расставляют коэффициенты в уравнении реакции, соответствующие числу электронов, участвующих в процессе. Далее уравнивают число атомов, не участвующих в окислении-восстановлении: число атомов металлов, не изменивших С.О.; ионы кислотных остатков, не изменивших С.О.; число атомов водорода. Проверка правильности расстановки коэффициентов осуществляется подсчетом общего числа атомов кислорода слева и справа.

                    4 Br20 +  H2+1S-2  + 4 H2+1O-2 → 8H+1Br -1  +  H2+1S+6O4-2

Примеры решения задач

Пример 1. Определить степень окисления ванадия в молекуле Na4V2O7.

Р е ш е н и е. Суммарный заряд молекулы равен нулю. Атомы Na и O относятся к элементам с постоянной степенью окисления (табл. 3). Учитывая, что в молекуле Na4V2O7  4 атома натрия и 7 атомов кислорода, составляем уравнение:

4 (+1) + 2x + 7 (-2) =0,  при решении которого находим, что степень окисления ванадия в указанном соединении равна +5:  x = +5.

Пример 2. Определить степень окисления кремния в ионе SiO44–.

Р е ш е н и е. В этом случае суммарный заряд соединения равен –4. Постоянная С.О. кислорода равна –2 (табл.3). Учитывая что атомов кислорода в ионе SiO44– четыре, составляем и решаем уравнение: x+ 4 (-2) = –4,   x = +4.

Пример 3. Определить, окислением или восстановлением являются данные переходы:              а) H2O2  O2 ,           б) MnO4  MnO2.

Р е ш е н и е. а)  Расставляем степени окисления элементов, входящих в состав соединений: . Данный переход представляет процесс окисления, так как происходит отдача электронов ионами кислорода:     2O–1 – 2e-  O2

б) Расставляем степени окисления элементов, входящих в состав соединений: . Данный переход представляет процесс восстановления, так как происходит присоединение электронов ионами марганца:      Mn+7 + 3e Mn+4.

1.12. Электрохимические процессы в гетерогенных системах.

Гальванические элементы

В отличие от гомогенных реакций, протекающих во всем объеме системы, гетерогенные взаимодействия осуществляются на границе раздела фаз.

Процессы взаимного превращения химической и электрической форм энергии называются электрохимическими процессами.

Рассмотрим процессы, протекающие при погружении металла в воду или раствор соли данного металла, в качестве примера гетерогенного взаимодействия.

Так как совершенно нерастворимых веществ не существует, то любой металл хотя бы в ничтожной степени  растворяется в воде. При этом в раствор переходят положительно заряженные ионы металла Меn+, а в самом металле остаются электроны, в результате чего металл заряжается отрицательно. Уравнение, отражающее  процесс ионизации атомов металла под действием полярных молекул воды имеет вид:

Ме0+ m H2O [Me(H2O)m]n+  + ne           (29)

в растворе       в кристалле Ме

Катионы, перешедшие в жидкость, располагаются вблизи поверхности отрицательно заряженной пластинки металла, в результате чего образуется двойной электрический слой. На границе раздела раствора соли с металлом возникает определенная разность потенциалов, которая называется электродным потециалом ЕМе/Меn+ [В].

Абсолютное значение электродного потенциала измерить невозможно. Поэтому измеряют разность потенциалов между данным электродом и стандартным водородным электродом, потенциал которого условно принимают равным нулю: Е02Н+/Н2 = 0В.  Электродный потенциал, измеренный по отношению к стандартному водородному электроду в стандартных условиях (Т = 250C (298 К), Р = 1 атм, СМеn+ = 1 моль/л), называется стандартным электродным потенциалом  (обозначается Е0).

Зависимость электродного потенциала от природы металла, температуры и концентрации ионов металла выражается уравнением Нернста:

Е=Е0 + (RT/nF)lg CMen+,         (30)

Где  (RT/F)=0,059  при Т=298 К. Тогда уравнение принимает вид:

ЕМеn+/Me0=Е0 Меn+/Me0 + (0,059/n)lg CMen+,      (31)

Е0 Меn+/Me0 – стандартный электродный потенциал Ме;

n – число электронов, участвующих в электродном процессе;

CMen+ - концентрация ионов металла, моль/л (молярная концентрация).

В зависимости от величины Е0 Меn+/Me0 все металлы можно расположить в ряд, который называется ряд стадартных электродных потенциалов металлов (табл. 4). В этом ряду (слева направо) величина электродного потенциала возрастает, при этом восстановительная способность металлов уменьшается, а окислительная способность ионов металлов  увеличивается.

Таблица 4

Ряд стандартных электродных потенциалов металлов

Восст.форма

Li

K

Na

Mg

Zn

Fe

Ni

Pb

H2

Cu

Ag

Au

Окисл.форма

Li+

K+

Na+

Mg2+

Zn2+

Fe2+

Ni2+

Pb2+

2H+

Cu2+

Ag+

Au3+

ЕМе/Меn+, В

-3,04

-2,92

-2,71

-2,36

-0,76

-0,44

-0,25

-0,13

0

+0,34

+0,80

+1,50

Этот ряд характеризует химические свойства металлов:

Металлы, обладающие более отрицательным электродным потенциалом, способны вытеснять менее активные металлы (т.е. имеющие более положительные электродные потенциалы) из растворов их солей.

Металлы, имеющие более отрицательный электродный потенциал по  отношению   к    водородному,  вытесняют водород из кислот – HCl (разб. и конц.) и H2SO4 (разб.), так как окислителем в них является ион H+.

По восстановительной активности все металлы делят на три группы:

1)  активные металлы: Е0 Меn+/Me0<  -1В;

2)  металлы средней активности:     -1В < Е0 Меn+/Me0 <0 В;

3) малоактивные металлы: Е0 Меn+/Me0 >0 В.

В концентрированной серной кислоте H2SO4 (конц.) роль окислителя выполняет анион SO42- за счет иона S+6, который является более сильным окислителем, чем H+.  Взаимодействие металлов с ней протекает по схеме:

 Me0 + H2S+6O4 (конц.) =  Me2+n (SO4)n  +  H2O + S+4O2 – для малоактивных металлов;

Me0 + H2S+6O4 (конц.) =  Me2+n(SO4)n  +  H2O +S0 – для металлов средней активности;

Me0 + H2S+6O4 (конц.) =  Me2+n(SO4)n  +  H2O + Н2S–2 – для активных металлов.

В азотной кислоте любой концентрации окислителем является анион NO3- за счет иона N+5. При взаимодействии HNO3 (конц.) с металлами она восстанавливается до иона N+4,  образуя газ NO2, независимо от активности металла. При взаимодействии HNO3 (разб.) с металлами взаимодействие протекает по схеме:

Me0 + HN+5O3 (разб.) =  Me+n (NO3)n  +  H2O + N+2O – для малоактивных металлов;

Me0 + HN+5O3(разб.)= Me+n (NO3)n+  H2O+ N2+1O – для металлов средней активности;

Me0 + HN+5O3(разб.)= Me+n (NO3)n+  H2O + N-3H4NO3 – для очень активных металлов.

Гальванический элемент (ГЭ) – это устройство, в котором энергия химической реакции превращается в электрическую. В основе работы гальванического элемента лежит ОВР. Гальванический элемент состоит из двух металлических  электродов (анода и катода): пластин, изготовленных из разных металлов и погруженных в растворы своих солей. ГЭ имеет две цепи: внешнюю – металлический проводник, соединяющий пластины, по которому двигаются электроны от анода к катоду, и внутреннюю – электролитический ключ, соединяющий растворы солей, по которому двигаются ионы в направлении от катода к аноду.

Максимальная разность потенциалов, которая может быть получена при работе гальванического элемента, называется электродвижущей силой гальванического элемента (ЭДС) – Еэдс [В]: Еэдс= Еок – Евос или   Еэдс= Ек – Еа.

Для рассмотрения электрохимического процесса, лежащего в основе работы любого гальванического элемента, следует использовать алгоритм, предложенный в примерах решения задач. Необходимо помнить, что в ГЭ анодом является металл с большей восстановительной активностью, т.е. с меньшим значением электродного потенциала, а катодом – металл с меньшей восстановительной активностью, т.е. с более высоким  значением электродного потенциала. При написании уравнений электродных процессов следует учитывать, что более активный металл является восстановителем, а ион менее активного металла – окислителем.

Примеры решения задач

Пример 1. Вычислите ЭДС и определите направление тока во внешней цепи данного гальванического элемента:

FeFeSO4AgNO3Ag, учитывая, что концентрация ионов Fe2+ и Ag+ соответственно равна 0,1моль/л и 0,01моль/л.

Р е ш е н и е. 1) Составляем схему гальванического элемента и указываем концентрацию ионов Fe2+ и Ag+  в растворах электролитов:

FeFeSO4AgNO3Ag

C Fe2+ = 10-1 моль/л и С Ag+ = 10-2 моль/л.

2) Пользуясь уравнением Нернста, вычисляем значения электродных потенциалов  железа и серебра в растворах заданной концентрации:

EFe2+/Fe0 = E0 Fe2+/Fe0 + (0,059/2) lg CFe2+ = – 0,44 + (0,059/2) lg10-1 = – 0,47B,

EAg+/Ag0 = E0 Ag+/Ag0 + 0,059) lg CAg+ = + 0,80  + 0.059  lg10-2 = + 0,68 В

EFe2+/Fe0  EAg+/Ag0, следовательно, более активным металлом является железо, оно будет отрицательным электродом – анодом, а серебро – катодом.

Таким образом,  Fe – анод (А) и Fe – восстановитель, Ag – катод.

3) Указываем направление движения электронов во внешней цепи, учитывая, что электроны движутся от анода к катоду:

(–)FeFeSO4AgNO3Ag(+)

4) Составляем электронные уравнения процессов, протекающих на электродах, учитывая, что на аноде происходит окисление атомов железа, а на катоде – восстановление ионов серебра:

(-) (А)  Fe0 – 2eFe2+         1– процесс окисления

(+) (K)  Ag+ + eAg0           2 – процесс восстановления

Fe0 +2 Ag+Fe2+ +2 Ag0

5) Записываем молекулярное уравнение окислительно-восстановительной реакции, лежащей в основе работы гальванического элемента:
                 
Fe0 + 2AgNO3Fe(NO3)2 + 2Ag0

6) Рассчитываем  ЭДС гальванического элемента:

Еэдс = ЕК – ЕА = E Ag+/Ag0E Fe2+/Fe0 = 0,68 – (–0,47) = 1,15 В.

Пример 2. Будет ли магний взаимодействовать с раствором сульфата никеля.

Р е ш е н и е. Для решения этой задачи необходимо сравнить стандартные электродные потенциалы магния и никеля:

E0Mg2+/Mg0 = – 2,34В,  E0Ni2+/Ni0  = – 0,25В

Магний – металл, имеющий более отрицательное значение стандартного электродного потенциала и поэтому являющийся более сильным восстановителем. Следовательно, магний будет подвергаться окислению под действием ионов никеля, и электроны от магния будут переходить к никелю:

Mg0 – 2eMg2+            

Ni2+ +2eNi0           

Mg0 + Ni2+ → Mg2+ + Ni0

Mg + NiSO4 = MgSO4 + Ni

1.13. Коррозия металлов

Коррозия – разрушение металла под воздействием окружающей среды.

Это самопроизвольный окислительно-восстановительный процесс, протекающий на границе раздела фаз. По механизму протекания коррозия подразделяется на химическую (протекает в средах, не проводящих электрический ток) и электрохимическую (протекает в средах, проводящих электрический ток).

Основные причины электрохимической коррозии (ЭХК) наличие в металле примесей других металлов и контакт металла с другими металлами, отличающимися по активности. Согласно теории ЭХК при соприкосновении металла с раствором электролита на его поверхности возникает множество гальванических микроэлементов. При этом анодами являются частицы основного металла, катодами – примеси, с большим значением электродного потенциала.

Одной из особенностей электрохимической коррозии является ее многостадийность. Рассмотрим процесс коррозионного разрушения металла на примере коррозионного гальванического элемента, возникающего при контакте железа и меди: Fe  электролит  Cu. Для того, чтобы понять, какой из этих двух металлов будет подвергаться коррозии, необходимо сравнить значения их стандартных электродных потенциалов: E0Fe2+/Fe0 = – 0,44В,  E0 Cu2+/Cu0  = + 0,34В.

E0Fe2+/Fe0  E0 Cu2+/Cu0, следовательно, железо является более активным восстановителем:  Fe – анод (А),  Cu – катод (К).

На первой стадии происходит окисление более активного металла (анодный процесс) и переход образовавшихся ионов в раствор:       Fe0 – 2e- =Fe2+ 

Вторая стадия – перенос электронов от анода к катоду, который при этом  заряжается отрицательно, т.е. поляризуется.

На третьей стадии происходит процесс восстановления (катодный процесс), в котором участвует окислитель окружающей среды. Он “забирает” электроны у катода, т.е. снимает с него отрицательный заряд и, таким образом, деполяризует катод. Процесс отвода электронов с катода называется деполяризацией, а окислитель – деполяризатором.

Важнейшими окислителями, вызывающими ЭХК, являются ионы водорода и  растворенный в воде молекулярный кислород. В связи с этим различают два вида электрохимической коррозии: с водородной  и с кислородной деполяризацией.

Электрохимическая коррозия с водородной деполяризацией протекает в кислой среде. Коррозионному разрушению подвергаются металлы, удовлетворяющие условию:   Е0Меn+/Ме0  Е0 +/Н20 (Е0 +/Н20 =0).

Электрохимическая коррозия с кислородной деполяризацией протекает в  нейтральной (влажный воздух, морская вода, влажные почвы) или щелочной средах. Коррозионному разрушению подвергаются металлы, стандартный электродный потенциал которых меньше стандартного электродного потенциала кислорода:
Е0Меn+/Ме0  Е0 О2 /2Н2О         (Е0 О2 /2Н2О = 1,23В).

При рассмотрении механизма электрохимической коррозии следует использовать алгоритм, приведенный в примерах решения задач.

Для защиты металлов от коррозии используют различные виды защитных покрытий, в том числе металлические покрытия. Анодное покрытие – покрытие основного металла более активным металлом, т.е. Е0осн.Ме  Е0покр.Ме  (например, покрытие железа цинком). Катодное покрытие – покрытие основного металла менее активным, т.е. Е0осн. Ме   Е0 покр. Ме  (например, покрытие железа никелем).

Примеры решения задач

Пример 1. Алюминий находится в контакте с цинком. Какой из этих металлов будет окисляться, если эта пара попадет в кислую среду, например, в среду соляной кислоты?

Р е ш е н и е. Из условия задачи следует что металлы находятся в кислой среде – растворе HCl. Раствор HCl – электролит, т.е. электропроводящая среда, следовательно, будет протекать электрохимическая коррозия. Для рассмотрения механизма коррозии воспользуемся предложенным выше алгоритмом.

1) Составим схему коррозионной гальванопары:

AlHClZn

2) Укажем окислитель. Среда кислая, поэтому окислителем (деполяризатором) является ион водорода H+. Следовательно, в этой схеме будет протекать электрохимическая коррозия с водородной деполяризацией.

3) Определим, какой из металлов будет являться анодом, а какой – катодом. Для этого сравним значения стандартных электродных потенциалов алюминия и цинка:
           
Е0Al3+/Al0= – 1,6 B < E0Zn2+/Zn0= – 0,77 B,

Значит, алюминий – более активный металл, он является восстановителем и анодом, а цинк – катодом:    Al – анод (А), Zn – катод (К).

4) Укажем направление движения электронов, учитывая, что электроны движутся от анода к катоду, а от катода – к окислителю  окружающей среды:

(-) AlHClZn(+)

 

5) Запишем электронные уравнения процессов, протекающих на электродах, и составим суммарное уравнение:

(–)(A) Al0 – 3eAl3+         2

(+)(K) 2Н+ + 2е → Н2↑      3

2Al0 +6H+ → 2Al3+ + 3H2

6) Составим молекулярное уравнение окислительно-восстановительной реакции, протекающей при коррозии:             2Al + 6HCl → 2AlCl3 + 3H2

7) Запишем вывод: при коррозии алюминия, находящегося в контакте с цинком, окисляется алюминий. Продуктом его коррозии является соль – хлорид алюминия. На цинковом катоде выделяется водород.

Пример 2. Изделие из меди с оловянным покрытием находится во влажном воздухе. Какой из металлов будет корродировать при нарушении целостности покрытия? К какому типу покрытий относится в этом случае олово?

Р е ш е н и е. Изделие находится во влажном воздухе, который является электропроводящей средой, следовательно, будет протекать электрохимическая коррозия.

1) Составим схему коррозионного  гальванического элемента:

SnH2OCu

2) Укажем окислитель. Вода – это нейтральная среда, поэтому окислителем (деполяризатором) является кислород – О2. Следовательно, в этой схеме будет протекать электрохимическая коррозия с кислородной деполяризацией.

3) Определим, какой из металлов будет являться анодом, а какой – катодом. Для этого сравним значения стандартных электродных потенциалов олова и меди:

Е0Sn 2+/ Sn0= – 0,14 B  <  E0Cu2+/Cu0= + 0,34 B.

Значит, олово – более активный металл, оно является восстановителем и анодом, а медь – катодом:  Sn – анод (А),  Cu – катод (К).

4) Укажем направление движения электронов, учитывая, что электроны движутся от анода к катоду, а от катода – к окислителю среды:

(–) SnH2OCu (+)

5) Запишем электронные уравнения процессов, протекающих на электродах, и составим суммарное уравнение. При написании уравнения катодного процесса следует учитывать, что процесс восстановления протекает в присутствии воды:

(-)(A)     Sn0 – 2e → Sn2+                  2

(+)(K) O2 + 2H2O + 4е →4OН      1

2Sn0 +2O2 + 2H2O → 2Sn2+ +4OH

6) Составим молекулярное уравнение окислительно-восстановительной реакции, протекающей при коррозии:   2Sn0 +2O2 + 2H2O → 2 Sn(OH)2

7) Запишем вывод: по отношению к меди олово является анодным покрытием, так как в этой паре олово выступает в роли анода. При нарушении целостности покрытия корродировать будет олово. Продуктом его коррозии является основание – гидроксид олова.

Пример 3. Медная деталь разрушается в атмосфере кислорода при температуре 2000С.  В чем заключается причина этого явления?

Р е ш е н и е. 1) Определяем характер среды: атмосфера кислорода (О2) при высокой температуре (2000С) – это неэлектропроводящая среда. Следовательно, будет происходить химическая коррозия.

2) Запишем уравнение процесса, протекающего при химической коррозии медной детали:

Cu0 + O20 = 2CuO

Вывод: происходит окисление меди и на поверхности детали образуется оксидная пленка.

1.14. Электролиз

Электролиз – это совокупность окислительно-восстановительных процессов, протекающих на электродах при пропускании постоянного электрического тока через раствор или расплав электролита. При электролизе происходит превращение электрической энергии в химическую энергию.

Электролиз проводят в электролизере, который представляет собой емкость, заполненную раствором или расплавом электролита, в который погружены два электрода (например, графитовые) – катод и анод. Катод подсоединяют к отрицательному полюсу внешнего источника тока, а анод – к положительному. Таким образом, катод – отрицательно заряженный электрод, а анод – положительно заряженный электрод. На катоде происходит процесс присоединения электронов катода  (от внешнего источника тока) катионами, атомами, молекулами.  На аноде происходит процесс отдачи электронов аноду (во внешнюю цепь)  анионами, атомами, молекулами. Таким образом, на катоде протекает процесс восстановления, на аноде – процесс окисления.

Рассмотрим механизм электролиза на примере расплава соли  фторида натрия NaF. Расплавы солей являются сильными электролитами, поэтому при высоких температурах полностью диссоциируют на ионы в соответствии с уравнением процесса:               NaF = Na+ + F.

Под действием электрического тока ионы  приобретают направленное движение: катионы будут двигаться к катоду, а анионы – к аноду. Достигнув электродов, ионы начнут разряжаться на их поверхности. На катоде будет происходить процесс восстановления ионов Na+, а на аноде – процесс окисления ионов F. Запишем электронные уравнения этих процессов и просуммируем их:

):  Na+ + eNa0        2  – процесс восстановления

(A+):  2F- – 2eF2          1  – процесс окисления

2Na+ + 2F-Na0 + F2

Суммарное молекулярное уравнение электролиза:

Электролиз водных растворов электролитов осложняется участием в электродных процессах молекул воды. Алгоритм рассмотрения электролиза водных растворов солей приведен в примерах решения задач.

При электролизе водных растворов электролитов необходимо учитывать особенности катодных и анодных процессов:

1. Катионы металлов от Li+ до Al3+ включительно (Е0Меn+/Ме0 < – 1,66 В) не восстанавливаются на катоде, вместо них на катоде восстанавливаются молекулы воды в соответствии с уравнением реакции:   2О + 2е = Н2 + 2ОН    

2. Катионы металлов  от Mn2+ до H, восстанавливаются на катоде вместе с молекулами воды:     Меn+ + ne = Me0 ;                 2О + 2е = Н2 + 2ОН    

3. Катионы металлов от Cu2+ до Au3+ полностью восстанавливаются на катоде в соответствии с уравнением реакции:       Меn+ + ne Me0

Если водный раствор электролита содержит катионы различных металлов, то при электролизе выделение их на катоде протекает в порядке уменьшения значений стандартных электродных потенциалов металлов.

4. На практике используют два типа анодов: инертные или нерастворимые (уголь, графит, золото, платина) и активные или растворимые (Cu, Ag, Ni, Zn и др). Инертные электроды не участвуют в процессе электролиза. Активные аноды сами участвуют (окисляются) в процессе электролиза в соответствии с уравнением реакции:              Ме0ne = Men+.

5. Анионы кислородсодержащих кислот (SO42-, NO3-, CO32-, PO43-), в которых центральный атом (S+6, N+5, C+4, P+5).находится в высшей степени окисления, а также ионы F никогда не окисляется на аноде из водных растворов электролитов. Вместо них окисляются молекулы воды:     2О – 4е = О20 + 4Н+

Количественные соотношения  между массой выделившихся на электродах веществ и количеством прошедшего электричества через раствор или расплав электролита выражают законом Фарадея:

  ,       (32)

где m – масса вещества, выделившегося на электроде;

Э – его химический эквивалент;

Q – количество электричества (Кл);

F – число Фарадея (F= 96500 Кл).

Q = I·t,  где I – сила тока (А),      t –  время электролиза (с).

    Э=А/n,  где А – атомная масса элемента; n – степень окисления его в соединении.

Тогда выражение (31) примет вид:

      (33)

При электролизе в реальных условиях масса выделившегося вещества (его практический выход) всегда меньше массы вещества, рассчитанной по закону Фарадея. Это объясняется тем, что наряду с основными электродными процессами протекают побочные  процессы, на которые расходуется часть количества электричества. Поэтому вводится  понятие   «выход по току»:

Впт  =mпр/mтеор×100%,          (33)

где mпр – масса вещества, практически полученного в процессе электролиза;

     mтеор - масса вещества, теоретически рассчитанная по закону Фарадея.

Примеры решения задач

Пример 1. Составить схему электролиза нитрата свинца с угольными электродами.

Р е ш е н и е. 1) Определим состав электролита. Для этого запишем уравнение электролитической диссоциации  соли Pb(NO3)2:

Pb(NO3)2 = Pb2+ + 2NO3

(nH2O)  (nH2O)

То есть в растворе присутствуют гидратированные ионы Pb2+и   NO3, а также молекулы воды, представляющие собой диполи Н+– ОН-.

2) Составляем условную схему электролиза, на которой показываем распределение ионов в пространстве и ориентацию полярных молекул воды у поверхности электродов. При прохождении тока через раствор положительно заряженные ионы будут двигаться к катоду, отрицательно заряженные – к аноду; молекулы воды будут определенным образом ориентироваться у поверхности электродов: положительно заряженным полюсом диполя у катода, а отрицательно заряженным полюсом – у анода. Таким образом, у поверхности каждого из электродов  на электродах будут скапливаться два сорта частиц: у катода – катионы Pb2+ и молекулы воды, а у анода – анионы NO3 и молекулы воды. Следовательно, на каждом из электродов будут протекать два конкурирующих процесса.

3) На основании сравнения значений стандартных электродных потенциалов определяем частицы, участвующие в катодном процессе (восстановления):
Е
0Pb2+/Pb0 = – 0,13 B > Е0Н2О/Н2= –0,41 (–0,83) В, следовательно, ионы Pb2+ обладают большей окислительной активностью и поэтому они участвуют в катодном процессе в соответствии с уравнением  реакции:  Pb2+ + 2e = Pb0

4) Определяем частицы, участвующие в анодном процессе (окисления).

Вспоминаем, что нитрат-ион относится к числу кислородсодержащих ионов, в котором центральный атом (N+5) находится в высшей степени окисления, поэтому ион  NO3не будет окисляться на аноде, вместо него окисляются  молекулы воды:
             
2О – 4е = О20 + 4Н+

5) Составляем суммарное ионное, а затем – суммарное молекулярное уравнения процесса электролиза:

(K):  Pb2+ + 2ePb0         2

(A+):  2О – 4е = О20 + 4Н+          1

2Pb2+ +  2Н2О = 2Pb0 + О20 + 4Н+ 

2Pb(NO3)2 +  2Н2О = 2Pb0 + О20 + 4НNO3

Таким образом, при электролизе водного раствора Pb(NO3)2 с угольными электродами в процессе электролиза участвует как сам электролит, так и молекулы воды. Основными продуктами электролиза являются атомы свинца и молекулярный кислород, побочным продуктом является азотная кислота, накапливающаяся в прианодном пространстве.

Пример 2. Раствор содержит ионы Fe2+, Ag+, Bi3+, и Pb2+ одинаковой концентрации. В какой последовательности будут разряжаться эти катионы при электролизе, если напряжение достаточно для выделения любого металла?

Р е ш е н и е. При электролизе катионы разряжаются на катоде, где происходит процесс восстановления. Восстанавливаются ионы, обладающие окислительной активностью, которая тем выше, чем больше значение стандартного электродного потенциала. Следовательно, необходимо сравнить значения стандартных электродных потенциалов заданных ионов металлов:

Е0Fe2+/Fe0 = – 0,44В,  Е0Ag+/Ag0 = 0,8В,  Е0Bi3+/Bi0 = 0,21В,   Е0Pb2+/Pb0 = – 0,13В

Сравниваем эти значения: Е0Ag+/Ag0  >  Е0Bi3+/Bi0  >  Е0Pb2+/Pb0  >  Е0Fe2+/Fe0. Следовательно, в ряду Ag+Bi3+Pb2+Fe2+ окислительная активность уменьшается, поэтому легче всего восстанавливаются ионы Ag+, затем ионы Bi3+, далее – ионы Pb2+ и в последнюю очередь – ионы Fe2+. Восстановление ионов Fe2+ будет происходить совместно с молекулами воды.

Запишем электронные уравнения процессов, протекающих на катоде:

Ag+ + eAg0 ,    Bi3+ + 3eBi0 ,    Pb2+ +2 ePb0  ,        Fe2+ + 2eFe0

Пример 3. Написать уравнения реакций, протекающих на электродах при электролизе водного раствора хлорида никеля с никелевым анодом.

Р е ш е н и е. 1) Определяем состав электролита. Записываем уравнение электролитической диссоциации хлорида никеля:  NiCl2 = Ni2+ + 2 Cl

(nH2O)  (nH2O)

2) Составляем условную схему электролиза (аналогичную схеме в задаче
№ 1), на которой показываем распределение ионов в пространстве и ориентацию полярных молекул воды у поверхности электродов. К катоду подходят катионы
Ni2+ и молекулы воды, ориентированные положительным полюсом к поверхности катода; к аноду подходят анионы Cl и молекулы воды, ориентированные отрицательным полюсом к поверхности анода.

3) Определяем, какие частицы будут участвовать в катодном процессе на основании сравнения значений стандартных электродных потенциалов:

(K)   Е0Ni2+/Ni0 = – 0,25 B,   Е0Н2О/Н2 = – 0,41 (–0,83)В

Е0Ni2+/Ni0 > Е0Н2О/Н2, следовательно, ионы Ni2+ обладают большей окислительной активностью и поэтому они участвуют в катодном процессе в соответствии с уравнением реакции:    Ni2+ + 2e = Ni0

4) Определяем, какие частицы будут участвовать в анодном процессе. Для этого выписываем значения стандартных электродных потенциалов трех частиц:  Cl-, H2O и Ni, поскольку материал анода также является химически активным:

+):  Е0Cl2/2Cl- = + 1,36 В,   Е0О2 / 2Н2O = + 1,23 ÷ (1,8)В ,  Е0Ni2+/Ni0 = – 0,25 B.

Сравнение значений Е0 показывает, что   Е0Ni2+/Ni0 < Е0Cl2/2Cl-<  Е0О2 / 2Н2O, поэтому большей восстановительной активностью обладают атомы никеля. Следовательно, сам никелевый анод будет участвовать в процессе окисления:

Ni0 – 2e = Ni2+

5) Объединяем уравнения катодного и анодного процессов:

(K)  Ni2+ + 2e = Ni0

(А+)  Ni0 – 2e = Ni2+

При суммировании получается 0 = 0, что лишено физического смысла. Поэтому рассмотрение электролиза завершают на этом этапе.

Таким образом, электролиз водного раствора NiCl2 с никелевым анодом сводится к анодному окислению атомов никеля (Ni0) и катодному восстановлению ионов никеля (Ni2+), т.е. переносу никеля с анода на катод. При этом количество электролита в растворе остается неизменным.

1.15. Свойства и получение полимеров

Высокомолекулярные соединения (ВМС) – это химические вещества с большой молекулярной массой  и обладающие особыми свойствами.

Химия ВМС изучает вещества, молекулы которых состоят из огромного числа атомов, соединенных между собой ковалентными связями. Такие молекулы называют макромолекулами: 2Н4)nполиэтилен; (C6H10O5)nцеллюлоза; (C5H8)nнатуральный каучук. Высокомолекулярные соединения называют еще полимерами.

Макромолекулы большинства полимеров построены из одинаковых групп атомов. Эти одинаковые, многократно повторяющиеся группы атомов, связанные между собой ковалентными связями, называются элементарными звеньями. Элементарное звено, как правило, представляет собой остаток соответствующего мономера:

2Н4)n – полиэтилен – элементарное звено    –СН2–СН2–;

(C2H3Cl)nполивинилхлорид (ПВХ) –    –СН2–СНCl–;

(C5H8)nнатуральный каучук (полиизопрен)  –СН2–СН=С(СН3)–СН2

Поэтому для изображения полимеров используют суммарные формулы:

(–СН2–СН2–)n,  (–СН2–СНCl–)n ,  (–СН2–СН=С(СН3)–СН2–)n.

Высокомолекулярные соединения обладают следующими специфическими свойствами:

– более низкая скорость растворения (по сравнению с низкомолекулярными веществами);

– высокая вязкость (более высокая, чем вязкость концентрированных растворов мономеров);

– протекание процесса деформации во времени (вследствие медленного  проявления упругих свойств полимерных материалов);

– гибкость цепных макромолекул, обеспечивающая постоянное изменение их конфигурации;

– зависимость свойств макромолекул от их геометрической формы (линейные макромолекулы обладают высокой прочностью и эластичностью; разветвленные полимеры обладают большей растворимостью по сравнению с линейными; сетчатые полимеры не растворяются и не плавятся без разложения).

В зависимости от порядка элементарных звеньев в цепи различают регулярные и нерегулярные полимеры. В регулярных полимерах отмечается правильно повторяющееся расположение атомов в макромолекуле; в нерегулярных – хаотичное расположение атомов. Например, натуральный каучук – это полимер изопренаCH2=C(CH3)–CH=CH2, а синтетический каучук – полимер бутадиена-1,3 (CH2=CHCH=CH2).

Сополимеры – это полимеры, макромолекулярные цепи которых построены из разных элементарных звеньев, например, бутадиеннитрильный каучук получают сополимеризацией бутадиена-1,3 (CH2=CHCH=CH2) с акрилонитрилом (CH2=CHCN).

Существуют следующие способы получения полимеров: метод полимеризации и метод поликонденсации. Полимеризация – это реакция соединения молекул мономера, протекающая за счет разрыва кратных связей и не сопровождающаяся выделением побочных низкомолекулярных продуктов. В реакцию полимеризацию вступают, главным образом, ненасыщенные соединения, у которых кратные связи находятся между атомами углерода:

n CH2CHCl     [–CH2–CHCl–]n

винилхлорид  ПВХполивинилхлорид

n H2CO        [–CH2O–]n

формальдегид  полиметиленоксид

Поликонденсация – это процесс образования полимеров путем химического взаимодействия молекул мономеров, сопровождающийся выделением низкомолекулярных веществ (H2O, NH3, HCl, C2H5OH и др.):

HOOC–(CH2)4–COOH   +        HNH–(CH2)6–NH2      →

  адипиновая кислота            гексаметилендиамин

→     HOOC–(CH2)4–CO–NH–(CH2)6–NH2(CH2)6–NH2    +        H2O

  гексаметиленадипинат

По физическому состоянию все полимеры делятся на:

текучие – необратимо меняют форму под действием даже незначительных нагрузок (например, полиизобутилен) и имеют аморфное строение;

высокоэластичные (эластомеры) – обратимо деформируются под действием небольших нагрузок; имеют аморфное строение (каучук, резина);

твердые – мало изменяют свою форму даже при больших нагрузках; после устранения нагрузки способны восстановить свою форму.

По отношению к воздействию тепла полимеры бывают

термопластичными (способны размягчаться при нагревании и снова затвердевать при охлаждении, сохраняя свои свойства: растворимость, плавкость);

термореактивными (способны размягчаться при нагревании, а при затвердевании –  переходить в неплавкое и нерастворимое состояние).

Все полимеры подвергаются деструкции. Деструкция – это процесс разрушения полимеров, протекающий с разрывом связей основной макромолекулярной цепи, приводящим к понижению молекулярной массы полимера. Деструкция бывает следующих видов:

химическая – протекает под действием химических реагентов (воды, спиртов, аммиака, кислот, щелочей и др.); например, кислотный гидролиз крахмала с образованием глюкозы:

6Н10О5)n + Н2О  + H+nC6H12O6;

механическая – происходит при действии механических напряжений (происходит изменение структуры и свойств полимера, связанное с разрывом макромолекулы);

– окислительная – протекает под действием окислителей, например, кислорода;

термическая – происходит при нагревании полимеров;

фотохимическая – происходит под действием света (= 300 – 400 нм) и сопровождается разрывом полимерной цепи;

радиационная – происходит под влиянием нейтронов, а также -, -,
- излучения. Разрываются связи С–С, С–Н и образуются низкомолекулярные продукты;

– биологическая – протекает под действием микроорганизмов (например, нитрат целлюлозы, поливинилацетат, натуральный каучук).

2.  КОНТРОЛЬНЫЕ ЗАДАНИЯ

2.1. Строение атома

  1.  Запишите электронную конфигурацию атома ванадия в основном состоянии и все квантовые числа его неспаренных электронов. Сколько свободных d-орбиталей содержится на предпоследнем энергетическом уровне? Какие химические свойства проявляет атом ванадия?
  2.  Запишите электронные конфигурации атомов элементов с зарядами ядер +20 и +35.  Покажите графически распределение электронов на внешних уровнях  и определите химические свойства  атомов этих элементов.
  3.  На основании электронных конфигураций атомов Са и S укажите, какой из этих элементов проявляет металлические, а какой - неметаллические свойства и почему?
  4.  Сколько энергетических уровней имеет атом, если для его внешних электронов
    главное квантовое число
    n = 3? Составьте полную электронную формулу строения атома галлия, укажите строение его внешнего уровня и изобразите его электронную схему. Какими химическими свойствами обладает атом галлия?
  5.  Запишите электронные конфигурации ионов Сr3+ и S2-. Напишите электронные уравнения процессов образования этих ионов из нейтральных атомов. Какие свойства при этом проявляют атомы Сr  и  S?
  6.  Определите, какой химический элемент содержит 32 электрона? Напишите  электронную конфигурацию атома этого элемента, составьте электронную схему строения его внешнего уровня   и укажите, какими химическими свойствами обладает атом этого элемента?
  7.  Что определяет положение Са и Вr в 4-м периоде периодической системы? Дайте объяснение, исходя из электронного строения внешних энергетических уровней атомов этих элементов. Почему Са  находится во II группе, а Вr - в VII группе?
  8.  Запишите электронные конфигурации атома мышьяка и его иона As+3 . Напишите электронное уравнение процесса образования этого иона из нейтрального атома. Какое свойство при этом проявляет атом мышьяка?
  9.  Напишите электронные конфигурации атомов элементов по указанным координатам в периодической системе: а) 3-й период, IA-группа; б) 4-й период, VA-группа. Изобразите схемы распределения электронов незавершенных подуровней. Определите химические свойства этих атомов.
  10.  Составьте электронные формулы атомов Аl и Мn. Выделите валентные электроны и изобразите графически их распределение по aтомным орбиталям. Какими химическими свойствами обладают атомы алюминия и марганца?

2.2. Периодический закон

  1.  На основании положения кремния в периодической системе элементов напишите его электронную формулу и формулы высшего оксида, гидроксида и хлорида.
    1.  В качестве примера назовите химические элементы, являющиеся электронными аналогами.  Почему они так называются? Составьте электронные конфигурации атомов этих элементов и дайте ответ.
    2.  Запишите электронные конфигурации ионов Sn+2 и Sn+4. Объясните, как соотносятся энергии  ионизации у этих ионов и их радиусы?
    3.  Сравните химические свойства атомов элементов с порядковыми номерами 12, 13 и 16. Составьте формулы их высших оксидов и соответствующих им гидроксидов. Охарактеризуйте кислотно-основные свойства этих оксидов и гидроксидов. Как они изменяются в ряду указанных элементов?
    4.  Составьте формулы оксидов и гидроксидов элементов 3-го периода периодической системы, отвечающих их высшей степени окисления. Как изменяются кислотно-основные свойства этих соединений при переходе от Na к С1?
    5.  У какого из р-элементов V группы периодической системы – фосфора или сурьмы – сильнее выражены неметаллические свойства? Почему? Какое из водородных соединений данных элементов является более сильным восстановителем?
    6.  Как изменяются свойства химических элементов, простых веществ и их соединений (оксидов и гидроксидов) в главных подгруппах? Разберите на примере элементов главной подгруппы V группы.
    7.  Какой из атомов  − хлор или йод проявляет окислительные свойства при образовании молекулы ICl из атомов? У какого из этих атомов сильнее выражена способность притягивать к себе электроны? Какие заряды (степени окисления) имеют хлор и йод в молекуле ICl?
    8.  Элементы кремний и титан расположены в одной группе периодической системы. Составьте электронные формулы атомов этих элементов и скажите, можно ли считать их электронными аналогами? Ответ поясните.
    9.  Как влияет повышение степени окисления элемента на свойства его гидроксидов? Ответьте на вопрос: какой из двух гидроксидов является более сильным основанием: СuОН или Сu(ОН)2; Fe(OH)2 или Fe(OH)3?

2.3.  Химическая связь

  1.  Составьте электронные формулы строения атомов водорода, хлора и фтора. На основании строения внешнего энергетического уровня этих атомов приведите схемы перекрывания электронных облаков при образовании молекул F2 и НС1. Какая из этих молекул образована ковалентной полярной связью?
    1.  Определите количество химических связей, которое может образовывать атом селена в нормальном и возбужденном состояниях. Дайте ответ, исходя из электронной формулы и электронной схемы строения атома селена.
      1.  Приведите примеры молекул, в которых реализуются следующие типы химических связей: а) ковалентная неполярная; б) ковалентная полярная; в) ионная. Ответ объясните.
      2.  Определите  максимальную валентность кислорода, фтора и серы, составив   электронные схемы строения внешних энергетических уровней  атомов этих элементов.
      3.  Определите химические свойства атома мышьяка, его валентность и возможные степени окисления в основном и возбужденном состояниях. Сколько химических связей в молекуле AsH3?
      4.  Определите возможные валентности атома серы. В основном или возбужденном состоянии находятся атомы серы  при образовании молекул H2S и SO3?
      5.   Как изменяются степень полярности и длина химической связи в молекулах галогеноводородов (Н–Г, где Г – Cl, Br, J) при последовательном переходе от HCI к HJ?
      6.  В основном или возбужденном состоянии находятся атомы фосфора и мышьяка при образовании молекул РС13 и РС15 и  AsH3 и H3AsО4?
      7.  Определите, в каком из оксидов элементов 3-го периода периодической системы связь Э–О наиболее приближается к ионной?
      8.  Распределите молекулы S2, MgO, HF,CO в порядке возрастания полярности связи. Ответ подтвердите расчетом.

2.4. Классы неорганических соединений

  1.  Определите, с какими из перечисленных веществ реагирует азотная кислота: CaO,  Сr2О3, NaOH,  Р2О5, СаСОз, СО2. Напишите уравнения возможных реакций.
    1.  К какому типу солей относятся: питьевая сода NaHCO3 , кальцинированная сода Na2CO3, двойной суперфосфат Са(Н2РО4)2, малахит uОН)2СО3? Назовите эти вещества по систематической номенклатуре.
      1.  На каких свойствах гидроксидов алюминия и железа основан способ очистки оксида алюминия от оксида железа (III) в бокситах при производстве алюминия путем обработки бокситов щелочью, отделения осадка и обработки полученного раствора соляной кислотой? Напишите уравнения реакций.
        1.  Из перечисленных оксидов: СаО, ZnO, SO2, Fe2O3, CO, CuO, SiO2, Al2O3  выпишите формулы: а) основных оксидов; б) кислотных оксидов; в) амфотерных оксидов. Составьте формулы соответствующих им гидроксидов.
        2.  Напишите уравнения реакций возможных способов получения  сульфата кальция, исходя из генетического ряда двух веществ: серы и кальция.
        3.  Определите, с какими из перечисленных соединений будет взаимодействовать оксид серы (IV): NaOH, Н3РО4, Н2О, Сr2О3, MnSO4, Ве(ОН)2, H2SO4 и КС1. Составьте уравнения соответствующих реакций и назовите получаемые продукты.
        4.  Какие из перечисленных веществ будут взаимодействовать с раствором гидроксида натрия: Р2О5, СО2, А1(ОН)3, MgO, H2SO4. Составьте уравнения соответствующих реакций и назовите получаемые продукты.
        5.  Какие новые соединения могут быть получены, если в Вашем распоряжении имеются следующие вещества: оксид кальция, вода, соляная кислота, углекислый газ и гидроксид бериллия? Напишите уравнения соответствующих реакций.
        6.  С какими из перечисленных соединений: КОН, HNО3, Na2O, СО2, Н2О  будет взаимодействовать оксид алюминия? Напишите уравнения возможных реакций и назовите получаемые продукты.
  2.  Составьте молекулярные уравнения реакций, с помощью которых можно осуществить следующие превращения:

Са(ОН)2 — СаСОз — Са(НСО3)2 — СаСО3 —СО2 —(NH4)2CO3.

2.5. Основы термохимии

  1.  Используя значения стандартных теплот образования соединений, вычислите тепловой эффект реакции образования 171 г сульфата алюминия из оксида алюминия и серы (VI).
  2.  Напишите термохимическое уравнение эндотермической реакции восстановления 12,76 г оксида меди (II) углеродом (до образования оксида углерода (II)) и вычислите теплоту образования оксида меди (II), если тепловой эффект процесса равен 8,24 кДж.
  3.   При восстановлении алюминием 480 г Fe2O3 выделяется теплота
    ∆Н0298= –2567 кДж. Зная, что ∆Н0f,298 Al2O3 = –1677 кДж/моль, рассчитайте тепловой эффект реакции:   Fe2O3 + 2A1 = А12О3 + 2Fe,  ∆Н < 0 и энтальпию образования Fe2O3.
  4.  Найдите стандартную теплоту образования NH3 (г), зная, что окисление NH3 протекает по термохимическому уравнению

4NH3(г) + ЗО2(г) = 2N2(г) + 6Н2О(г) ,  ∆Н°298 = – 1530 кДж .

  1.  Кристаллический хлорид аммония образуется при взаимодействии газообразных NH3 и НС1. Приведите термохимическое уравнение этой реакции, вычислив ее тепловой эффект.
  2.  Сколько теплоты выделится при сгорании серы массой 240 г, если известно, что изменение энтальпии образования оксида серы (IV) из кислорода и серы равно
    − 297 кДж/моль?
  3.  Вычислите, сколько литров азота (н.у.) участвовало в реакции с водородом при образовании аммиака, если при этом выделилось 18,45 кДж теплоты?
  4.  Вычислите энтальпию образования Fe2O3, если в реакции
    Fe2O3(т) + ЗСО(г) = 2Fe(т) + ЗСО2(г), изменение энтальпии  ∆Н°298 = −28,4 кДж ,
    а энтальпии образования участников реакции имеют следующие значения:

∆Н°f,298 СО2(г) = −393,0 кДж/моль,  ∆Н0 f, 298 СО (г), = − 110,5 кДж/моль.

  1.  Вычислите теплоту образования гидроксида кальция, исходя из следующих термохимических уравнений:

Са(т) +1/2О2(г) = СаО(т) , ∆Н°298 =− 635,1 кДж ;

Н2(г) + 1/2О2(г) = Н2О(ж) , ΔН°298 =−285,8 кДж ;

СаО(т) + Н2О(ж) = Са(ОН)2(т), ΔН°298 = − 45,0 кДж.

  1.  Рассчитайте, выделением или поглощением теплоты сопровождается термическое разложение хлорида аммония? Напишите термохимическое уравнение данной реакции.

2.6. Основы химической термодинамики

  1.  Возможно ли осуществление процесса восстановления металлического железа из оксида железа Fe2O3  действием водорода при стандартных условиях.
  2.  Как изменяются основные свойства оксидов элементов IIА-группы? Ответ подтвердите расчетом ∆G°298 реакций взаимодействия оксидов металлов с диоксидом углерода (IV) для СаО и MgO : ЭО(т) + СО2(г) = ЭСО3(т), где Э – Са , Mg.
  3.  Вычислите значения ∆Н0298, ∆S0298, ∆G0298 для процесса МеСОз(т)= МеО(т) + СО2(г)

и сравните термическую стабильность карбонатов MgCO3 и СаСО3.

  1.  Предскажите знак изменения энтропии в следующих реакциях и проверьте предсказание расчетами: а) СаО(т) + СО2(г) = СаСОз(т),
                                               б) 
    А12О3(т) + 3SO3(г) = A12(SO4)3(т).
  2.  Вычислите изменение энергии Гиббса и определите возможность процесса получения титана магнийтермическим способом – восстановлением хлорида титана (IV) металлическим магнием при 1000 К.
  3.  Определите, при какой температуре возможен самопроизвольный процесс восстановления оксида железа (III) по реакции

Fe2O3(т) + ЗСО(г) = 2Fe(т) + ЗСО2(г), ∆Н°298 = 346 кДж .

  1.   Какие из приведенных водородных соединений могут быть получены в стандартных условиях непосредственно из элементов, а какие – косвенным путем: Н2О(г), H2S(г), H2Se(г) и Н2Те(г)? Вывод обоснуйте сравнением значений ∆G0298 реакций  Н2 + Э = Н2Э, где Э – О , S , Se, Те.
  2.  Можно ли использовать реакцию Сr2О3(т) + ЗС(т) = 2Сr (т) + ЗСО(г) для получения хрома при 1500К? Изменение энергии Гиббса определите на основании расчета теплового эффекта и изменения энтропии этой реакции.
  3.  Определите, при какой температуре в реакционной системе

4НС1(г) + О2(г) 2О(г) + 2С12(г) , ∆H0298= − 114,4 кДж,

наступит термодинамическое равновесие. Приведите соответствующие расчеты, принимая во внимание, что в состоянии равновесия реализуется условие ∆G0 = 0.

  1.  Определите, прямая или обратная реакция будет протекать при стандартных условиях и при 500К в системе 2NO(г) + О2(г)  2NO2(г). Ответ обоснуйте, вычислив значения ∆G0298 и ∆G0500 прямой реакции.

2.7. Химическая кинетика

  1.  Рассчитайте, как изменится скорость реакции образования оксида азота

N2(г) + О2(г)     2NO(г), если общее давление в системе увеличить в 3 раза.

  1.  При 20°С реакция протекает за 2 мин. За сколько времени будет протекать эта реакция: а) при 0° С; б) при 50°С? Температурный коэффициент скорости равен 2.
  2.  Рассчитайте, как изменится скорость прямой и обратной реакций в гомогенной системе 2SO2 + О2  2SO3, если уменьшить объем, занимаемый газами, в два раза? Сместится ли при этом равновесие системы?
  3.  Процесс окисления аммиака протекает по уравнению

4NH3(г) + 5О2(г) = 4NO(г) + 6Н2О(г).

Определите, как изменится скорость прямой реакции: а) при увеличении давления в системе в два раза; б) при уменьшении концентрации аммиака в 3 раза.

  1.  Известно, что при 25°С две реакции протекают с одинаковой скоростью. Определите соотношение скоростей этих реакций при 95°С, если известно, что температурный коэффициент скорости первой реакции равен 2, а второй равен 3.
  2.  Рассчитайте, как и во сколько раз изменится скорость химической реакции
    2Cu2О (т)+ О2(г) = 4СuО (т)  а) при повышении температуры на 30°С (γ = 3); б) при уменьшении парциального давлении кислорода в  газовой фазе в 2 раза.
  3.  В реакции Na2S2O3 + 2HC1 = 2NaCl + S + SO2 + Н2О

исходные концентрации реагирующих веществ равны CNa2S2O3 =1 моль/л и СHCl =
2 моль/л. Как изменится скорость реакции, если концентрацию тиосульфата натрия увеличить до 3 моль/л, а концентрацию соляной кислоты до 6 моль/л?

  1.  Во сколько раз увеличится скорость прямой реакции Н2 + Сl2 2HCl при повышении температуры от 25°С до 200°С, если известно, что при повышении температуры на каждые 25° С скорость этой реакции увеличивается в четыре раза?
  2.  С увеличением температуры на 10°С скорость химической реакции увеличивается в 2 раза. Определите скорость реакции при 40°С и 10°С, если известно, что при 20°С скорость реакции равна 0,08 моль/л • ч.
  3.  Реакция при температуре 50°С протекает за 2 мин 15 сек. За сколько времени закончится эта реакция при температуре 70°С, если в данном температурном интервале температурный коэффициент скорости равен 3?

2.8. Химическое равновесие

  1.  Укажите и объясните, в каком направлении будет смещаться равновесие в системах:     2СО(г) СО2 (г) +  С (т),       ∆H0298= –173,0 кДж

                   MgCO3(т) MgO(т)+СО2(г),  ∆H0298  = 385,0 кДж

а) при повышении температуры; б) при понижении общего давления. Напишите выражения для константы равновесия этих реакций.

  1.  Вычислите константу равновесия для обратимой реакции, протекающей по уравнению 2NО2(г) 2NO(г)+О2(г), если известно, что в состоянии равновесия
    [NO2] = 0,06 моль/л , [NO] = 0,24 моль/л и [О2] = 0,22 моль/л. В каком направлении сместится равновесие при повышении давления?
  2.  Напишите выражение для константы равновесия следующих обратимых реакций:

2НВr(г) Н2(г) + Вr2(г) ,  ∆H0298= 72,5 кДж;

3Fe(т) + 4Н2О(г) Fe3O4(т), + 4Н2(г) ,    ∆H0298 = –152,3 кДж .

Объясните, как повлияют увеличение концентрации реагирующих веществ и понижение температуры на химическое равновесие в этих системах.

  1.  При сгорании метана протекает экзотермическая реакция

СН4(г) + 2О2(г) СО2(г) + 2Н2О(г)

Напишите выражение для константы равновесия этой системы. Какие внешние воздействия надо оказать на систему для увеличения выхода углекислого газа?

  1.  При некоторой температуре константа равновесия Кс гомогенной системы
    N2 + О2 2NO  равна 4,1•10 -4. Вычислите равновесную концентрацию О2, если равновесные концентрации N2 и NO соответственно равны 0,10 и 0,05 моль/л.
  2.  Начальные концентрации NO, Н2 и Н2О в гомогенной системе
    2NO + 2Н2  N2+ Н2О соответственно равны 0,10; 0,05 и 0,10 моль/л. Вычислите равновесные концентрации Н2, N2 и Н2О, если равновесная концентрация
    [NO] = 0,07 моль/л. Чему равна константа равновесия?
  3.  При некоторой температуре  константа равновесия Кс гомогенной системы
    N2+ 3H2  2NH3  равна 0,1. Равновесные концентрации  Н2 и  NH3 соответственно равны 0,2 и 0,08 моль/л. Вычислите равновесную и начальную концентрацию N2.
  4.  Как влияет на равновесие гетерогенной системы С(т) + СО2(г) ⇄ 2C0(г), ∆Hº298 = +119,8кДж: а) добавление СО2(г); б) добавление С(т); в) повышение температуры; г) повышение давления; д) введение катализатора; е) удаление СО(г)?
  5.  Эндотермическая реакция разложения пентахлорида фосфора протекает по уравнению   РС15(г) РС13(г) + С12(г),  ∆Hº298= +92,6 кДж.

Как надо изменить: а) температуру; б) давление; в) концентрацию, чтобы сместить равновесие в сторону прямой реакции разложения РС15?

  1.  Напишите выражения  константы равновесия Кс для равновесных систем:
    а)
    СaCO3(т)    CaO(т)+СО2(г);    б)  СО(г)+Н2О(г) CO2(г)+Н2(г)

От каких факторов зависит константа равновесия?

2.9. Способы выражения концентрации растворов

  1.  Вычислите молярную концентрацию 16%-го раствора хлорида аммония, плотность которого равна 1,15 г/см3.
  2.  В 1 л водного раствора содержится 577г H2SO4. Плотность раствора –
    1,34 г/см
    3. Вычислите процентную и молярную  концентрации этого раствора.
  3.  Вычислите процентную концентрацию 8 М раствора HNO3 (плотность равна 1,25 г/см3).
  4.  К 100 мл 96%-го раствора H2SO4  (плотность раствора равна 1,84 г/мл) прибавили 400 мл воды. Получился раствор с плотностью, равной 1,225 г/мл. Чему равна процентная концентрация полученного раствора?
  5.  Определите молярную концентрацию раствора, полученного при растворении сульфата натрия массой 42,6 г в воде массой 300 г, если плотность полученного раствора равна 1,12 г/мл.
  6.  Плотность 5,5 М раствора NaOH равна 1,2 г/мл. Определите массовую долю
    (в процентах) гидроксида натрия в этом растворе.
  7.  В воде массой 200 г растворили гидроксид калия массой 11,2 г. Плотность полученного раствора равна 1,04 г/мл. Рассчитайте молярную концентрацию этого раствора.
  8.  Вычислите молярную концентрацию хлороводородной кислоты в растворе с массовой долей НС1 20% (плотность раствора равна 1,1 г/мл).
  9.  В воде массой 1000 г растворили 2 моль сульфата калия. Вычислите массовую долю растворенного вещества.
  10.  Чему равна молярная концентрация 30%-го раствора NaOH , плотность которого составляет 1,33 г/мл? Вычислите процентную концентрацию раствора, полученного добавлением 5 л воды к 1 л 30%-го раствора NaOH.

2.10. Электролитическая диссоциация

  1.  Напишите уравнения электролитической диссоциации солей, применяемых в качестве фосфорных удобрений: Са(Н2РО4)2 , СаНРО4 , К3РО4. Назовите эти соли.
  2.  Напишите уравнения электролитической диссоциации следующих электролитов: А1С13, H2SO3, HC1, Na2HPO4, Cu(OH)2. Перечислите особенности процессов диссоциации сильных и слабых электролитов.
  3.  Определите степень диссоциации синильной кислоты HCN в 1М растворе, если константа диссоциации равна 7,2•10-10. Вычислите концентрацию ионов Н+ и CN  в этом растворе.
  4.  Степень диссоциации 0,1М раствора угольной кислоты по первой ступени равна
    2,11•10
    –3. Вычислите соответствующую константу диссоциации этой кислоты.
  5.  Определите, при какой концентрации раствора степень диссоциации азотистой кислоты будет равна 0,2, если известно, что константа диссоциации  Кд = 4•10–4.
  6.  В 0,1 М растворе степень диссоциации уксусной кислоты СН3СООН равна
    1,32•10
    -2. Определите, при какой концентрации азотистая кислота будет иметь такую же степень диссоциации, если константа диссоциации азотистой кислоты равна 4,6•10-4.
  7.  Вычислите степень диссоциации и концентрацию ионов Н+ и HS–- в 0,1 М растворе H2S , если константа диссоциации по первой ступени равна 3•10-7.
  8.  Вычислите концентрацию ионов водорода в 0,02 М растворе сернистой кислоты при константе диссоциации по первой ступени, равной 1,3•10-2. Диссоциацией кислоты по второй ступени пренебречь.
  9.  Вычислите степень диссоциации и концентрацию ионов NH4+ и ОН в 0,5 М растворе 4,OH, если константа диссоциации нашатырного спирта составляет
    1,8∙10
    -5.
  10.  Как изменится степень диссоциации плавиковой кислоты (HF), если
    а) раствор нагреть; б) раствор разбавить водой; в) к раствору добавить
    KF?

2.11. Реакции  ионного обмена

  1.  Раствор поваренной соли, предназначенный для электролиза, освобождают от ионов кальция и магния с помощью карбоната натрия. Составьте молекулярные и ионные уравнения этих реакций.
  2.  Составьте молекулярные и ионные уравнения реакций между водными растворами: а) карбоната натрия и серной кислоты; б) силикатом натрия и хлористоводородной кислоты; в) йодида калия и нитрата свинца (II).
  3.  Составьте два различных уравнения в молекулярной форме, которые будут соответствовать данному уравнению в сокращенной ионной форме: Ni2+ + S2– = NiS.
  4.  Составьте молекулярные и ионные уравнения реакций между водными растворами: а) сульфида калия и соляной кислоты; б) сульфата железа (II) и гидроксида натрия; в) нитрата никеля (II) и хлорида натрия.
  5.  Реакции в растворах выражаются следующими ионными уравнениями:

а) + + СО32– = СО2 + H2O;  б) Ni2++S2–=  NiS;  в) Н + + ОН = Н2О.

Составьте молекулярные уравнения этих реакций и объясните порядок их составления.

  1.  Напишите в молекулярной и ионной формах уравнения реакций взаимодействия между следующими водными растворами: а) хлорида железа (III) и гидроксида калия;  б) нитрата меди (II) и сульфита натрия; в) хлорида кальция и йодида натрия.
  2.  Можно ли приготовить растворы, содержащие одновременно следующие пары веществ: а) ZnCl2 и NaOH;   б) AgNO3 и Mg(CH3COO)2?

Составьте молекулярные и ионные уравнения возможных реакций.

  1.  Составьте молекулярные и ионные уравнения реакций взаимодействия между следующими веществами: а) нитратом бария и сульфатом натрия; б) карбонатом натрия и серной кислотой.
  2.  Перечислите условия необратимого протекания реакций ионного обмена. Приведите примеры, запишите уравнения реакций в молекулярной и ионной формах.
  3.  Выразите молекулярными уравнениями реакции, проходящие в растворах между следующими веществами: а) хлористым кальцием и углекислым аммонием;

б) азотнокислым кальцием и содой. В чем заключается сущность этих реакций? Каким одним ионным уравнением они могут быть выражены?

2.12. Гидролиз солей

  1.  Объясните образование гидроксида алюминия в результате протекания реакции:              A12(SO4)3 + ЗСа(НСО3)2 = 3CaSО4 + 2А1(ОН)3 + 6СО2.
  2.  Укажите, какую среду (кислую или щелочную) будут иметь вследствие гидролиза растворы солей сульфата меди (II) и хлорида аммония. Напишите уравнения соответствующих реакций.
  3.  Почему раствор Na2S имеет щелочную, а раствор ZnSO4 – кислую среду? Ответ подтвердите ионно-молекулярными и молекулярными уравнениями.
  4.  В какую сторону сместится равновесие гидролиза NaCN, если к раствору добавить: а) щелочь; б) кислоту?
  5.  Какую окраску приобретает фенолфталеин в растворе ацетата натрия CH3COONa? Почему окраска при нагревании усиливается, а при охлаждении ослабевает? Составьте ионно-молекулярное и молекулярное уравнения гидролиза этой соли.
  6.  К растворам Na2SO4, CrCl3, Fе(NО3)3 добавили раствор соды 2СО3. В каких случаях наблюдается выделение СО2? Почему? Составьте ионно-молекулярное и молекулярное уравнения соответствующих реакций.
  7.  В какой цвет будет окрашен лакмус в водных растворах NH4C1, Na2CO3 и NaNO3? Ответ обоснуйте, составив уравнения реакций гидролиза данных солей.
  8.  Почему при приготовлении водного раствора ZnCl2 его подкисляют соляной кислотой, а в водный раствор 2СО3 при его приготовлении добавляют гидроксид натрия?
  9.  Какие из солей: CaS, NaNO3 или FeCl3 подвергаются гидролизу? Почему? Составьте ионные и молекулярные уравнения гидролиза соответствующих солей по первой ступени. Каков характер среды водных растворов этих солей?
  10.  Гидролиз карбоната натрия описывается уравнением

Na2CO3 +H2O      NаНСОз + NaOH,          ∆H>0

Предложите способы увеличения гидролиза соли.

2.13. Коллоидные растворы

  1.  При пропускании избытка сероводорода H2S в раствор хлорида мышьяка AsCl3 получили золь сульфида мышьяка. Учитывая условия образования, напишите формулу мицеллы золя и определите знак его заряда.
  2.  Золь кремниевой кислоты H2SiO3 был получен при взаимодействии растворов K2SiO3 и HCl. Определите, какой из электролитов был в избытке, если противоионы в электрическом поле движутся к катоду. Напишите формулу мицеллы золя.
  3.  Составьте формулу мицеллы золя гидроксида алюминия, полученного при глубоком гидролизе сульфата алюминия Al2(SO4)3.
  4.  Какой объем 0,008 М раствора AgNO3 надо прибавить к 0,025 л 0,016 М раствора KI, чтобы получить отрицательно заряженные частицы золя иодида серебра. Напишите формулу мицеллы.
  5.  Золь бромида серебра получен путем смешивания равных объемов 0,008 М раствора KBr и 0,009 М раствора AgNO3. Определите знак заряда частиц золя и напишите формулу мицеллы.
  6.  Какой объем 0,001 М раствора FeCl3 надо прибавить к 0,03 л 0,002 М раствора AgNO3, чтобы частицы золя хлорида серебра в электрическом поле двигались к аноду? Напишите формулу мицеллы золя.
  7.  Золь гидроксида железа получен смешиванием равных объемов  0,002 М раствора NaOH и 0,0003 М раствора Fe2(SO4)3. Какой знак заряда имеют частицы золя? Составьте формулу мицеллы.
  8.  К 0,02 л 0,001 М раствора AsCl3 добавили 0.02 л 0,003 М раствора Na2S. Определите заряд частиц полученного золя и напишите формулу его мицеллы.
  9.  Какой объем 0,0025 М раствора KI надо добавить к 0,035 л 0,003 М раствора Pb(NO3)2, чтобы получить отрицательно заряженные частицы золя иодида свинца? Напишите формулу мицеллы золя.
  10.  Золь гидроксида меди получен при сливании 0,1 л 0,05 М раствора NaOH и 0,25 л 0,001 М раствора Cu(NO3)2. Определите заряд частиц полученного золя и напишите формулу его мицеллы.

2.14. Растворы неэлектролитов

  1.  Раствор, содержащий 3,04 г камфоры С10Н16О в 100 г бензола С6Н6, кипит при 80,70С. Температура кипения бензола 80,2 0С. Вычислите эбулиоскопическую константу бензола.
  2.  Вычислите осмотическое давление раствора, в литре которого содержится
    0,2 моль неэлектролита при 18
    0С.
  3.  На сколько градусов повысится температура кипения воды, если в 100 г воды растворить 9 г глюкозы C6H12O6? Эбулиоскопическая константа воды 1,86 0С.
  4.  Вычислите температуру кипения 5%-ного раствора нафталина C10H8 в бензоле C6H6. Температура кипения бензола 80,20; эбулиоскопическая константа 2,57 0С.
  5.  При 0 0С осмотическое давление раствора сахара С12Н22О11 равно 3,55105 Па. Какая масса сахара содержится в 1 литре раствора?
  6.  Понижение давления пара над раствором, содержащим 0,4 моль анилина C6H5NH2 в 3,04 кг сероуглерода CS2, при некоторой температуре равно 1007,7 Па. Давление пара сероуглерода при той же температуре 1,0133105 Па. Вычислите молекулярную массу сероуглерода.
  7.  Температура кипения раствора, содержащего 3,05 г бензойной кислоты C6H5COOH в 125 г хлороформа СН3Сl, равна 61,88 0С. Температура кипения хлороформа 61,12 0С. Вычислите эбулиоскопическую константу хлороформа.
  8.  Раствор, содержащий 2,56 г вещества в 500 г бензола С6Н6, кристаллизуется при 5,704 0С. Температура кристаллизации бензола 5,5 0С, криоскопическая константа 5,1 0С. Вычислите молярную массу растворенного вещества.
  9.  Вычислите температуру кристаллизации 5%-ного раствора этилового спирта C2H5OH, зная, что криоскопическая константа воды 1,86 0С.
  10.  В 0,5 л раствора содержится 2 г неэлектролита и раствор при 00С имеет осмотическое давление, равное 0,5110Па. Какова молекулярная масса неэлектролита?

2.15. Окислительно-восстановительные реакции

В задачах №141 - №150 методом электронного баланса подберите коэффициенты в уравнениях реакций. Укажите окислитель и восстановитель, процессы окисления и восстановления.

  1.  KMnO4 +Na2SO3 +H2О     →       Na2SO4+MnO2 + КОН,

PbS + HNO3                   Pb(NO3)2 + S + NO+ H2O

  1.  FeSO4 + КСlOз + H2SO4  → Fe2(SO4)3 + КС1 + Н2О,

Р + НJOз + Н2О   →  Н3РО4 + HJ.

  1.  NaCrO2 + РbО2 + NaOH  →  Na2CrO4 + Na2PbО2 + H2O,

J2 + NaOH  →  NaJ + NaJO + H2O.

  1.  K2Cr2О7 + H2S + H2SO4  → Cr2(SO4)3 + S + K2SO4 + H2О,

HNO2 → HNO3 + NO +H2O.

  1.  AgNO3 + Na2SO3 + NaOH  → Na2SO4 + Ag + NaNO3 + H2O,

KC1O3 → KC1O4 + KC1.

  1.  H3AsО3 +KMnO4 +H2SO4 → H3AsO4 + MnSO4+ K2SO4 + H2O,

CrCl3 + NaCIO + NaOH → Na2CrO4 + NaCl + H2O.

  1.  KMnO4 +NH3 → MnO2 + KNO3 + KOH + H2O,

MnO2 + HC1  → MnCl2 + Cl2 + H2O.

  1.  Fe2O3 + KNO3 + KOH → K2FeO4 + KNO2 + H2O,

КСlO3 + S → KC1 + SO2.

  1.  Na2S + KMnO4 + H2O  → S + MnO2 + NaOH + KOH,

Mn2O3 + Si  → SiO2 + Mn.

  1.  Na2S + Na2Cr2О7 + H2SO4 → S + Cr2(SO4)3 + Na2SO4 + H2O,

FeO + AlA12O3 + Fe.

2.16. Гальванические элементы

В задачах №151 – №160 приведены схемы гальванических элементов. Разберите работу гальванического элемента. Напишите электронные уравнения анодного и катодного процессов, составьте суммарное ионное уравнение окислительно-восстановительной реакции в гальваническом элементе и вычислите его ЭДС.

  1.  Zn | Zn(NO3)2 || Pb(NO3)2 | Pb

СZn2+ = 10-2 моль/л ,   CPb+2 = 1 моль/л

  1.  Сr | СrС13 || FeSO4 | Fe ,

СCr+3= 10 -3 моль/л ,   CFе+2 =10 -2 моль/л

  1.  A1 | A12(SO4)3 || CuSO4 | Cu,

СAl+3= 10–3 моль/л,  ССu+2 = 10 –2 моль/л

  1.  Mg | MgCl2 || BiCl3 |Bi

СMg+2 = 1 моль/л,   СBi+3 = 10 –3 моль/л.

  1.  Ni | Ni(NO3)2 ||AgNO3 | Ag,

CNi+2 = 10 -2 моль/л,  CAg+= 10 -1  моль/л

  1.  Ni | NiCl2 || AgNO3 | Ag,

CNi+2 = 10 -2 моль/л,  CAg+= 10 –1 моль/л

  1.  Mg | Mg(NO3)2 || Al2(SO4}3 | Al,

С Mg+2  =10-2 моль/л,   СAl +3 = 10-3 моль/л

  1.  Сu | Cu(NO3)2 || ZnSO4 | Zn,

ССu+2 = 1 моль/л, CZn+2 = 10-2 моль/л

  1.  Мn | MnSO4 || FeSO4 | Fe,

СMn+2 =10-2 моль/л ,   СFe+2 = 10-2 моль/л

  1.  Мn | МnС12 || NiCl2 | Ni,

СMn+2 = 10-2 моль/л,  СNi+2 = 10-2 моль/л

2.17. Коррозия металлов

  1.  Как протекает коррозия в случае повреждения поверхностного слоя оцинкованного и никелированного железа при их контакте с водой? Составьте схему коррозионного  гальванического элемента и укажите направление движения электронов. Напишите уравнения анодного и катодного процессов, а также суммарное уравнение электрохимической коррозии.
  2.  Алюминий склепан с медью. Какой из металлов будет подвергаться коррозии, если эти металлы попадут в кислую среду? Составьте схему коррозионного  гальванического элемента и укажите направление движения электронов. Напишите уравнения анодного и катодного процессов, а также суммарное уравнение электрохимической коррозии.
  3.  Олово спаяно с серебром. Какой из металлов будет окисляться при коррозии, если эта пара металлов попадет в щелочную среду? Составьте схему коррозионного гальванического элемента и укажите направление движения электронов. Напишите уравнения анодного и катодного процессов, а также суммарное уравнение электрохимической коррозии.
  4.  Какой металл в паре Fe-Ni будет растворяться в разбавленном растворе уксусной кислоты? На каком металле выделяется водород? Составьте схему коррозионного  гальванического элемента и укажите направление движения электронов. Напишите уравнения анодного и катодного процессов, а также суммарное уравнение электрохимической коррозии.
  5.  Составьте схемы двух коррозионных гальванических элементов, в одном из которых никель является катодом, а в другом – анодом. Приведите примеры и объясните сущность анодной и катодной защиты металлов от коррозии.
  6.  Какое покрытие металла называется анодным, а какое – катодным? Приведите несколько металлов, которые могут служить для анодного и катодного покрытия железа. Составьте схему коррозионного  гальванического элемента и укажите направление движения электронов. Напишите уравнения анодного и катодного процессов, а также суммарное уравнение электрохимической коррозии.
  7.  После длительного хранения на воздухе произошло полное разрушение цинкового изделия, контактирующего с медью. Объясните это явление. Составьте схему коррозионного  гальванического элемента и укажите направление движения электронов. Напишите уравнения анодного и катодного процессов, а также суммарное уравнение электрохимической коррозии.
  8.  Как происходит атмосферная коррозия луженого железа при нарушении целостности покрытия? Составьте схему коррозионного  гальванического элемента и укажите направление движения электронов. Напишите уравнения анодного и катодного процессов, а также суммарное уравнение электрохимической коррозии. Приведите уравнения реакций образования вторичных продуктов коррозии железа с учетом окисления гидроксида железа (II) до гидроксида железа (III), приводящего к образованию ржавчины.
  9.  Железные изделия, помещенные в свинцовый контейнер, при длительном хранении на воздухе подверглись коррозии. Объясните наблюдаемое явление. Составьте схему коррозионного гальванического элемента  и укажите направление движения электронов. Напишите уравнения анодного и катодного процессов, а также суммарное уравнение электрохимической коррозии.
  10.  Поясните, почему при никелировании железных деталей их предварительно покрывают медью, а потом никелем? Составьте электронные схемы процессов при коррозии никелированной детали, если слой никеля поврежден.

2.18. Электролиз водных растворов солей

  1.  В промышленности хлор получают электролизом концентрированного раствора хлорида натрия. Напишите уравнения электродных процессов, протекающих при электролизе этого раствора с угольными электродами.
    1.  Какой силы ток должен быть использован для того, чтобы выделить из раствора AgNO3 серебро массой 108 г за 6 минут? Составьте схему электролиза этого раствора при использовании графитовых электродов.
    2.  Какие процессы будут происходить на электродах при электролизе водного раствора хлорида меди (II): а) с графитовым; б) с медным анодом?
    3.  Через раствор сульфата железа (II) пропускали ток силой 13,4 А  в течение
      1
       часа. Определите массу железа, выделившуюся на катоде, если выход по току составляет 70%. Составьте схему электролиза этого раствора при использовании угольных электродов.
    4.  Какое количество электричества потребуется для получения 1 кг олова при электролизе водного раствора сульфата олова (II)? Напишите уравнения электродных процессов, протекающих при электролизе этого раствора с угольными электродами.
    5.  Составьте электронные уравнения процессов, протекающих на угольных электродах при электролизе водного раствора MgCl2. Вычислите силу тока, если известно, что при электролизе MgCl2 в течение 30 минут на катоде выделилось
      8,4 л водорода, измеренного при нормальных условиях.
    6.  Сколько времени необходимо проводить электролиз водного раствора хлорида золота (III) при силе тока 1 А для выделения на катоде 1 г золота? Приведите уравнения электродных процессов (анод угольный).
    7.  При электролитическом рафинировании меди через водный раствор сульфата меди (II ) пропускали ток силой 25 А в течение 4 часов. При этом на катоде выделилось 112 г меди. Рассчитайте выход по току. Составьте уравнения электродных процессов, учитывая, что анод изготовлен из меди, подлежащей очистке.
    8.  Покажите различие катодных реакций, происходящих при электролизе двух растворов – нитрата калия и нитрата серебра.
    9.  Какие реакции будут протекать на электродах при электролизе водного раствора нитрата никеля с инертными электродами и с никелевым анодом?

2.19. Химические свойства металлов

При решении задач №181 - №190 коэффициенты в уравнениях реакций подберите методом электронного баланса.

181. Медь не взаимодействует с разбавленной серной кислотой, но растворяется в концентрированной кислоте. В чем причина наблюдаемого явления? Составьте уравнение соответствующей реакции, а также напишите реакции взаимодействия меди с разбавленной и концентрированной НNО3.

182. С какими из перечисленных соединений взаимодействует магний: Н2О, NaOH, H2SO4(paзб.), H2SO4(конц.), HNO3(разб.), HNO3(конц.)? Напишите уравнения возможных реакций.

183. Составьте уравнения реакций взаимодействия никеля с соляной, разбавленной серной и азотной кислотами (при нагревании).

184. Напишите уравнения возможных реакций взаимодействия железа с кислотами:

НС1(конц.), H2SO4(разб.), H2SO4(конц.), HNO3(разб.), HNO3(конц.)? Почему в кислотах высокой концентрации (H2SO4 и HNO3) железо растворяется только при нагревании?

185. С какими из перечисленных соединений взаимодействует алюминий: Н2О,
НС1(конц.), H
2SO4(разб.), HNO3(разб.), NaOH. Напишите уравнения возможных реакций.

186. Составьте уравнения реакций взаимодействия серебра с разбавленной и концентрированной азотной кислотой. Объясните, почему серебро не взаимодействует с разбавленной серной кислотой, в то время как растворяется в концентрированной H2SO4. Напишите уравнение протекающей реакции.

187. Напишите уравнения химических реакций взаимодействия цинка с растворами: НС1, H2SO4(конц.), HNO3(разб.) и NaOH.

188. С какими из перечисленных кислот взаимодействует медь: НС1(конц.),
H2SO4(разб.), H2SO4 (конц.), HNO3(разб.).

189. Марганец находится в воде и контактирует с медью. Сохранятся ли оба металла в неизменном виде? Напишите электронные уравнения происходящих процессов.

190. Предложите способы перевода в раствор образцов сплавов латуни (массовая доля меди 69,5%, остальное - цинк) и бронзы (массовая доля меди 89,5%, остальное – олово) и способ разделения. Приведите уравнения соответствующих реакций.

2.20. Свойства и получение полимеров

191. Какие соединения относят к высокомолекулярным? Приведите классификацию полимеров по методам получения. Приведите схему получения полиметиакрилата.

192. Приведите основные методы получения высокомолекулярных соединений. Что такое сополимеры? Приведите схему получения бутадиеннитрильного каучука.

193. Что такое степень полимеризации? Почему вязкость полимеров выше по сравнению с растворами мономеров? Приведите схему получения поливинилацетата.

194. Перечислите основные способы полимеризации. Приведите схему получения полипропилена.

195. Что такое поликонденсация? В чем ее отличие от полимеризации? Приведите схему получения поликапроамида.

196. Чем объясняется медленная растворимость полимеров? Как получить синтетический каучук?

197. Приведите схемы получения а) изопренового каучука, б) поливинилового спирта.

198. Охарактеризуйте физические состояния полимеров: твердое, высокоэластичное, вязкотекучее. Приведите схему получения поливинилхлорида.

199. Что такое фотохимическая деструкция? Приведите схемы получения а) полистирола, б) полиакрилонитрила.

200. Что такое термопластичные и термореактивные полимеры? Приведите пример сетчатого полимера. Приведите схему получения ацетата целлюлозы.

3. ВАРИАНТЫ КОНТРОЛЬНЫХ ЗАДАНИЙ

Номер

вариан-та

Номер задания

1

2

3

4

5

6

7

8

9

10

00

1

31

51

81

91

101

131

141

151

161

01

2

32

52

82

92

102

132

142

152

162

02

3

33

53

83

93

103

133

143

153

163

03

4

34

54

84

94

104

134

144

154

164

04

5

35

55

85

95

105

135

145

155

165

05

6

36

56

86

96

106

136

146

156

166

06

7

37

57

87

97

107

137

147

157

167

07

8

38

58

88

98

108

138

148

158

168

08

9

39

59

89

99

109

139

149

159

169

09

10

40

60

90

100

110

140

150

160

170

10

11

21

41

61

71

111

131

151

161

181

11

12

22

42

62

72

112

132

152

162

182

12

13

23

43

63

73

113

133

153

163

183

13

14

24

44

64

74

114

134

154

164

184

14

15

25

45

65

75

115

135

155

165

185

15

16

26

46

66

76

116

136

156

166

186

16

17

27

47

67

77

117

137

157

167

187

17

18

28

48

68

78

118

138

158

168

188

18

19

29

49

69

79

119

139

159

169

189

19

20

30

50

70

80

120

140

160

170

190

20

21

51

71

100

101

121

140

160

170

171

21

22

52

72

99

102

122

139

159

169

172

22

23

53

73

98

103

123

138

158

168

173

23

24

54

74

97

104

124

137

157

167

174

24

25

55

75

96

105

125

136

156

166

175

25

26

56

76

95

106

126

135

155

165

176

26

27

57

77

94

107

127

134

154

164

177

27

28

58

78

93

108

128

133

153

163

178

28

29

59

79

92

109

129

132

152

162

179

29

30

60

80

91

110

130

131

151

161

180

30

10

30

40

61

80

110

120

150

161

181

31

9

29

39

62

79

109

119

149

162

182

32

8

28

38

63

78

108

118

148

163

183

33

7

27

37

64

77

107

117

147

164

184

34

6

26

36

65

76

106

116

146

165

185

35

5

25

35

66

75

105

115

145

166

186

36

4

24

34

67

74

104

114

144

167

187

37

3

23

33

68

73

103

113

143

168

188

38

2

22

32

69

72

102

112

142

169

189

39

1

21

31

70

71

101

111

141

170

190

40

20

21

41

51

90

111

131

160

170

190

41

19

22

42

52

89

112

132

159

169

189

42

18

23

43

53

88

113

133

158

168

188

43

17

24

44

54

87

114

134

157

167

187

44

16

25

45

55

86

115

135

156

166

186

45

15

26

46

56

85

116

136

155

165

185

46

14

27

47

57

84

117

137

154

164

184

47

13

28

48

58

83

118

138

153

163

183

48

12

29

49

59

82

119

139

152

162

189

49

11

30

50

60

81

120

140

151

161

181

50

10

20

31

70

71

121

140

160

170

171

51

9

19

32

69

72

122

139

159

169

172

52

8

18

33

68

73

123

138

158

168

173

53

7

17

34

67

74

124

137

157

167

174

54

6

16

35

66

75

125

136

156

166

175

55

5

15

36

65

76

126

135

155

165

176

56

4

14

37

64

77

127

134

154

164

177

57

3

13

38

63

78

128

133

153

163

178

58

2

12

39

62

79

129

132

152

162

179

59

1

11

40

61

80

130

131

151

161

180

60

20

21

31

51

90

110

111

140

150

160

61

19

22

32

52

89

109

112

139

149

159

62

18

23

33

53

88

108

113

138

148

158

63

17

24

34

54

87

107

114

137

147

157

64

16

25

35

55

86

106

115

136

146

156

65

15

26

36

56

85

105

116

135

145

155

66

14

27

37

57

84

104

117

134

144

154

67

13

28

38

58

83

103

118

133

143

153

68

12

29

39

59

82

102

119

132

142

152

69

11

30

40

60

81

101

120

131

141

151

70

21

41

51

81

100

121

140

141

151

171

71

22

42

52

82

99

122

139

142

152

172

72

23

43

53

83

98

123

138

143

153

173

73

24

44

54

84

97

124

137

144

154

174

74

25

45

55

85

96

125

136

145

155

175

75

26

46

56

86

95

126

135

146

156

176

76

27

47

57

87

94

127

134

147

157

177

77

28

48

58

88

93

128

133

148

158

178

78

29

49

59

89

92

129

132

149

159

179

79

30

50

60

90

91

130

131

150

160

180

80

20

21

40

61

91

101

131

141

160

170

81

19

22

39

62

92

102

132

142

159

169

82

18

23

38

63

93

103

133

143

158

168

83

17

24

37

64

94

104

134

144

157

167

84

16

25

36

65

95

105

135

145

156

166

85

15

26

35

66

96

106

136

146

155

165

86

14

27

34

67

97

107

137

147

154

164

87

13

28

33

68

98

108

138

148

153

163

88

12

29

32

69

99

109

139

149

152

162

89

11

30

31

70

100

110

140

150

151

161

90

1

21

51

90

110

111

140

150

160

170

91

2

22

52

89

109

112

139

149

159

169

92

3

23

53

88

108

113

138

148

158

168

93

4

24

54

87

107

114

137

147

157

167

94

5

25

55

86

106

115

136

146

156

166

95

6

26

56

85

105

116

135

145

155

165

96

7

27

57

84

104

117

134

144

154

164

97

8

28

58

83

103

118

133

143

153

163

98

9

29

59

82

102

119

132

142

152

162

99

10

30

60

81

101

120

131

141

151

161

ЛИТЕРАТУРА

Основная

  1.  Глинка Н.Л. Задачи и упражнения по общей химии: Учеб. пособие для вузов. М.: Интеграл-Пресс, 2011. 240 с.
  2.  Задачи и упражнения по общей химии: Учеб. пособие / Под ред.
    Н.В. Коровина. М.: Высшая школа, 2006. 249 с.
  3.  Коровин Н.В. Общая химия: Учеб. для вузов. М.: Высшая школа, 2009.
    557 с.

 Дополнительная

  1.  Ахметов Н.С. Общая и неорганическая химия: Учеб. для вузов. М.:  Высшая школа, 2009. 743 с.
  2.  Коржуков Н.Г. Общая и неорганическая химия: Учеб. пособие. М.: Миссис, 2004. 511с.
  3.  Угай Я.А. Общая и неорганическая химия: Учеб. для вузов. М.: Высшая школа, 2002. 527 с.

ЗАДАНИЯ И МЕТОДИЧЕСКИЕ УКАЗАНИЯ

К ВЫПОЛНЕНИЮ КОНТРОЛЬНЫХ РАБОТ

ПО ДИСЦИПЛИНЕ «ХИМИЯ»

Подписано в печать _________. Формат 6084/16. Бумага для множ. аппаратов.

Печать плоская. Усл. печ. л. ___. Уч.-изд. л.____. Тираж ____ экз. Заказ № ____.

ФГАОУ ВПО «Российский государственный профессионально-педагогический университет». Екатеринбург, ул. Машиностроителей, 11.

Ризограф ФГАОУ ВПО РГППУ. Екатеринбург, ул. Машиностроителей, 11.


Восст-ль

Окислительиььььь

ZnSO4

Zn

Сu

CuSO4

е-

SO42-

е-

e-

e-

e

К

А+

Расплав NaF

e

Na+

    F––

+

+

+

+

К

А+

Pb2+

Н+ОН

Pb2+

NO3

Н+ОН

NO3




1. 1] Кадровое планирование на предприятии
2. инновация интерпретируется как превращение потенциального научнотехнического прогресса в реальный вопл
3. те этого столкновения и образовались планеты в том числе и З
4. Вариант 14 В задачах 19 найти неопределённые интегралы ответ проверить дифференцированием
5. 3982 евро с чел. Rhodos Bech Hotel 3 BB 4170 грн с чел
6. Этуш Владимир Абрамович
7. где у богатых зауральских башкир
8. Политические преобразования в СССР 50-60е годы
9. тема обладает в каждый момент времени.
10. Особенности подбора и адаптации менеджеров по туризму среди выпускников ВУЗов
11. Бенуа АН
12. тема представлений и знаний человека о мире о самом себе и своих взаимоотношениях с окружающей действительн
13. Определение поражающих факторов
14. Пояснительная записка к расчетной работе по дисциплине Теоретические основы защиты окружающей среды
15. стилистический анализ
16. РЕФЕРАТ дисертації на здобуття наукового ступеня кандидата історичних наук Київ2003 Дисерта
17. Лекція 3 Болонський процес і перспективи розвитку вищої освіти в Україні
18. РЕФЕРАТ дисертація на здобуття наукового ступеня кандидата технічних наук Ха
19. КПІ Факультет електроніки Кафедра мікроелектроніки Протокол по вико
20. открывает книгу